Двач.hk не отвечает.
Вы видите копию треда, сохраненную вчера в 09:45.

Скачать тред: только с превью, с превью и прикрепленными файлами.
Второй вариант может долго скачиваться. Файлы будут только в живых или недавно утонувших тредах. Подробнее

Если вам полезен архив М.Двача, пожертвуйте на оплату сервера.
tumblroujih3a5ia1wxzu7qo1500.jpg52 Кб, 500x500
МАТЕМАТИКА ДЛЯ НАЧИНАЮЩИХ N+1 29047 В конец треда | Веб
В этом треде мы изучаем математику. Если ты школьник или студент, и у тебя есть трудности с задачей, то здесь тебе помогут её решить или хотя бы скажут, в каком направлении двигаться для её решения. Чем более чётко и конкретно ты опишешь суть своих затруднений, тем выше твой шанс на содержательный ответ.

Основные списки литературы:
http://pastebin.com/raw/4iMjfWAf - classic
http://pastebin.com/raw/4FngRj6n - dxdy

Архив тредов (там же остальные списки литературы и полезные ссылки):
https://pastebin.com/raw/qhs0WNbY
3 115918
>>5917
https://dmitripavlov.org/notes/topology.pdf

>These notes offer an elementary introduction to topology.


>нет определения топологии


Ты не обосрешься с определением топологии (как Вербит) если просто не будешь его давать, смекалочка.
4 115919
>>5918

>определением топологии


Картофан
5 115920
>>5918
подразумевается, что студент уже знаком с определением топологии и примерами вроде метрических пространств из курса анализа
сам курс про алгебраическую топологию и в этом плане выбранный им подход через симп. множества хорош и единственно правильный
verb-top.png171 Кб, 1363x1083
6 115921
>>5918
>>115852 →
Укажи, где в приведённом определении по твоему мнению ошибка. Скриншот взят из учебника топологии Вербицкого (МЦНМО, 2017. –352 с. ISBN 978-5-4439-1036-9)
7 115922
>>5921
Попробуй глаза разуть и почитать внимательнее.
>>5920

>подразумевается, что студент уже знаком


>>These notes offer an elementary introduction to topology.

1575196472711.png91 Кб, 1344x176
8 115923
>>5921
На пике должно быть $S \subseteq 2^M$, иначе $(M,S)$ не имеет смысла. Но это простая опечатка.
мимо
>>5922
Из контекста должно быть очевидно, что под "топологией" подразумевается "современная алгебраическая топология", и введение в нее действительно элементарное.
9 115924
>>5923
Вообще говоря я же не сомневаюсь что Вербит на самом деле знает правильное определение топологии. Просто никогда не устану угарать с такого эпичного обсера. Какого же говна понаписано во всех этих говнокнежонках, все время что то

>подразумевается


и надо догадываться что там у автора в голове и вообще лучше не браться за прочтение если ты не знаешь большую часть материала.
10 115925
>>5923

>Но это простая опечатка.


Я бы кстати не сказал что это опечатка. Скорее коекакерская небрежность, из разряда

>Из контекста должно быть очевидно


и кто надо то разберется.
11 115926
>>5923

>действительно элементарное


Критерий "элементарности"?
12 115927
>>5923
ок, согласен, $S$ не должно быть $\subset M$
при этом словами там всё написано правильно

>>5924

>с такого эпичного обсера


ты так называешь опечатку?
13 115928
>>5927
Еще не устану угарать с того какая же ты тупорылая ебанашка, но упорно не хочешь признавать себя таковой.
14 115929
>>5916 (Del)
Гипотезу о кобордизме доказал (прямо доказал, а не как Лурье), в обобщённом виде с геометрическими структурами тоже.
15 115930
>>5928
похоже, среди всего прочего петух-неосилятор не осилил определение топологии и это добавило отдельный пункт к его вонючему недовольству всем подряд
16 115931
>>5912 (Del)
>>5906 (Del)

>говнозадачи


Хорошая задача. Я вот сейчас, когда просыпался, вспомнил про неё и меня озарило, что показатель k по модулю k + 1 это всегда двоечка, потому что k = -1 (mod k+1), и, соответственно, имеем ситуацию, что первой степени мало, а вторая уже подходит.
1024 ответ.
17 115933
>>5931

>меня озарило


Так это и есть главный признак говнозадачи. А ты тупой наркоман словивший приход, поэтому неспособный мыслить рационально.
18 115934
>>5933
Через озарения математика и делается. Всё остальное — работа с ивзестным.
19 115935
>>5934
Тогда еще лучше вообще все учебники выкинуть и просто озарится всей математикой от древних греков до наших дней в одном мега-акте озарения.
20 115936
>>5935
Между двумя крайностями есть ещё целый спектр. Только замшелые консерваторы могут думать иначе.
21 115937
>>5936

>Только замшелые консерваторы могут думать иначе.


Тупой школотун, блядь.
22 115938
>>5937
Спокойствие, ретроград. Чекай современные достижения Западной академии и не будь баттхёртом.
23 115939
>>5938
Швятой жапад еще к чему то приплел, кашка.
24 115940
Аноны, посоветуйте задачник по мат физике.
Чтобы не сильно сложный, но ухватить самое главное и нужное
Можно на русском или англ
25 115941
$x=5.1^x-4.9^x$

Как решать?
26 115942
>>5940
одна из лучших книжек по классической части этого предмета - М.А. Шубин - Лекции об уравнениях математической физики

задач в ней немного, зато она довольно короткая (может показаться неполной, в зависимости от целей) и подготавливает выход в теорию пдо и микролокальный анализ, что здорово

если книга покажется трудной, я бы советовал Владимиров, Жаринов - Уравнения математической физики, где всё разжёвывается подробно

среди книг на англ у меня фаворитов нет
специализированные задачники мне неизвестны
27 115943
>>5942
Спасибо
Вторая выглядит норм
28 115944
>>5942

>Владимиров


>всё разжёвывается подробно


Мелкочмоха, ты сам то пробовал этого своего Владимирова открывать хоть раз? Чему из него научился интересно - в любой непонятной ситуации делай преобразование Фурье. Да? У него кстати и сборник задач есть - обрешайся хоть вусмерть. Странно что такой фанат (задачеблядства и Владимирова) не в курсе.
29 115945
>>5944
если тебе трудно тянуть Владимирова, то это только потому что ты петух-неосилятор, а не потому, что с ним что-то не так: ничего более элементарного и подробного из приличных текстов я не знаю

>преобразование Фурье


ты хотел сказать "джедайские техники", сначала ты именно так его называл
30 115946
>>5945
"Техники" учитывают граничные условия. Если ты расскажешь откуда в преобразовании Фурье берутся еще и граничные условия буду рад послушать.
Кстати, помнишь от чего у тебя так люто подгорело? Напомню - я написал что ты тупорылая чмоха и чтобы больше мне не писал. И это был еще ДО того как ты высрался искрометной техникой решения любого диф-уравнения:1) делаем ПФ 2) делаем обратное ПФ 3) решаем
31 115947
>>5946
обычно я рад что-нибудь рассказать, но тебе я рассказывать ничего не буду, само собой

про тот разговор у меня воспоминания другие, ты фантазируешь. решал я не дифференциальное уравнение; срач не останавливался, потому что ты ко мне лез, а я тебе честно отвечал (в точности, как сейчас: я ответил на посторонний вопрос, не твой, а ты полез и пытаешься затеять срач)
32 115948
>>5947
Конечно не будешь, это ж тебе не книжки нубасам советовать которые никогда не открывал, тут и хуев в рот напихать могут за гнилой пиздеж.
33 115949
>>5948
это очень грубая манипуляция - объяснять мой отказ тебе что-то рассказывать после всего, что ты мне писал

справедливости ради, этот вопрос достаточно глубоко исследуется (правда, это уже не базовая теория), и если тебе надо, ты можешь поискать самостоятельно.
34 115950
>>5949

>объяснять мой отказ


объяснять таким образом, я имел в виду
35 115951
>>5949

>можешь поискать самостоятельно


Вот это ты мне глаза открыл, мелкочмоха, чтобы я без тебя делал. Реально ведь этот опущ искренне считает что кому то помогает своими безмозглыми высерами.
36 115952
>>5951

>чтобы я без тебя делал


надо думать, то же, что ты делаешь всегда: ничего
37 115953
>>5952

>надо думать


Для тебя это неподъемная задача, чмоня.
38 115956
Почему в большинстве учебников рассказывают именно о правилах, но не объясняют почему так, а не иначе, я должен это интуитивно сам понять ?
39 115957
>>5956
это зависит от автора - считает он нужным давать мотивировки или нет и какие именно
40 115958
>>5957
Глубина анализа проблематики как всегда поражает, чмонь.
41 115959
>>5958
прихлопнуть бы тебя как муху, назойлив безобразно
42 115960
>>5956
Например?
Обычно все довольно интуитивно
43 115961
>>5960

>Например?


Любой школьный учебник

>Обычно все довольно интуитивно


Повезло тебе в жизни
44 115962
>>5961

>Повезло тебе в жизни


Так это ж типичный двачерский пиздабол. У него наверняка еще и хуй до колена и Ландау-Лифшиц хорошая книжка для новичков (хотя профессоры с мировым именем утверждают обратное почему то, неосиляторы наверное просто).
45 115963
>>5961

> Любой школьный учебник


Дурацкий пример, у меня их нет
Приведи пример правила, которое тебе кажется неинтуитивным/навязанным
46 115964
>>5956
А вообще интуиции всякие обычно прививают лекторы на лекциях или практики на практиках
47 115965
>>5964
Через жопоеблю?
48 115966
>>5956
Школьный учебник написан для массового использования. То есть для работы в условиях, когда престарелая карга с устаревшим на сто лет педагогическим образованием, со средним айсикью и зарплатой на 30% ниже средней по рынку симулирует процесс трансфера знаний к тридцати абсолютно бездарным ученикам из средних и низших слоев населения при затратах времени на уровне двух уроков в неделю (т. е. 12 минут на ученика в месяц).

Естественно, что подобные учебники пишутся в духе мантр, камланий, молитв и потрясаний конечностями в такт идеологическому и политическому бубну. "Завод зовет! Трудись на барина! Во имя ленина! Во славу сталина!", "У адольфа усики - у пиндосов трусики!", "С нами гугл и майкрософт, мы хуячим людям софт..." - и прочее, в зависимости от текущей экономической конъюктуры.

Государство - это мент, закаменевший в законе. А государственная школа - это скотомогильник by design. Даже если это номерная физматшкола, где каждый будущей гений находится на карандаше у гэбни и обязан защищать олимпиадное реноме колхоза, к которому он приписан. Просто наплюй на нее. При всей своей немыслимой вони она отменяется одним точным харчком.

По учебнику учится только скотина. Человек обучается из всех источников сразу.
49 115967
>>5963
Я дальше дробей не ушел поэтому они
Screenshot8.png46 Кб, 1152x245
50 115968
народы, а как доказать, что из d) следует e) и из e) следует a)?
51 115969
>>5967
Ну вот если у тебя есть несколько пицц, каждая из которых порезана на 3 куска и у тебя попросили 2/3 пиццы - это значит, что тебе нужно дать 2 куска
Если попросили 4/3 пицц - надо дать 4 куска
Если попросили 9/3 пицц - надо дать 9 кусков

Хотя лучше посмотри какие нибудь видосики на ютубе, там обычно хорошо объясняют
52 115973
>>5968

>e) => a)


подсказка: в предположении, что $f(x) = f(y)$ для некоторых $x \neq y$ рассмотреть $A = \{x,y\}$

>d) => e)


напрямую это немного неуклюже, но можно делать так:
1) достаточно показать $f(A \setminus B) \subset f(A) \setminus f(B)$
2) обозначая через $C = f(A \setminus B), \, D = f(A) \setminus f(B)$, выводим, что $C \setminus D \subset f(A) \cap f(X \setminus A) = \emptyset$, откуда 1)
53 115974
>>5956

>Почему в большинстве учебников рассказывают именно о правилах, но не объясняют почему так, а не иначе, я должен это интуитивно сам понять ?


потому что твой интерес не является тем фактором, который мог бы мотивировать автора учебной программы пускаться в пояснения. ты черточка в статистике, ради тебя не будут менять общую программу. а приводить углубленные объяснения перед общей аудиторией не считается подобающим.
54 115976
>>5974
3b1b же существует, почему бы всем не равняться на них?
55 115977
>>5973
Вот неясно, как всё-таки доказать, что $ C \setminus D \subset f(A) \cap f(X \setminus A) $.

А то, что $ f(A) \setminus f(B) \subset f(A \setminus B) $ это включение для любых множеств верно?

Вот моя попытка, дальше хз что делать.

Пусть
$x \in C \wedge x \notin D \Rightarrow x \in f(A\setminus B) \wedge x \notin (f(A) \setminus f(B)) \Rightarrow x \in f(A) \wedge \\ \ (\text{так как $f(A \setminus B) \subset f(A)$}) \wedge x \in (f(X) \setminus f(A) ) \cup f(B) \Rightarrow x \in f(A) \cap ((f(X) \setminus f(A) ) \cup f(B) )$.
56 115978
>>5977
вот моя вторая попытка. рассуждения верны?

по условию $B \cap A\setminus B = \varnothing \Leftrightarrow f(B) \cap f(A \setminus B) = \varnothing$. Пусть $x \in f(A \setminus B)$, тогда $x \notin f(B)$, и поскольку $f(A \setminus B) \subset f(A) \Rightarrow x \in f(A) \Rightarrow x \in f(A) \setminus f(B)$
58 115981
>>5980
Ну и говно
59 115982
>>5968

>d)


внутри самого d) следствие влево вытекает просто из определения отображения как однозначного отношения. и только следствие вправо нетривиально и связано с инъективностью.
инъективность [math]f[/math] означает, что одинаковые образы обязательно имеют одинаковые прообразы, и никогда не разные: [math]fx = fy \to x=y[/math]. или, иначе говоря, от противного: разные прообразы обязательно имеют разные образы: [math]x \neq y \to fx \neq fy[/math]
следствие вправо говорит, что если образы множеств неких прообразов не имеют общих образов, то сами множества этих прообразов не имеют общих прообразов. как это связано с инъективностью? говорить о различии [math]x[/math] и [math]y[/math]: [math]x \neq y[/math] - это то же самое, что говорить о них как об элементах неких непересекающихся множеств [math]A[/math] и [math]B[/math]. т. о. следствие вправо просто говорит о том, что разные прообразы имеют разные образы, но говорит это с помощью непересекающихся множеств.
конечно, нотация несимпатичная, смешивающая с одной стороны образы и прообразы, и с другой стороны множества образов и прообразов (которые, вообще говоря, могут сами быть среди образов и прообразов того же отображения). надеюсь, нотация нас не запутывает, и хотя я в общем против ее применения, но здесь ее использую.
далее, каким образом

>из d) следует e)


, имея в виду именно содержащееся внутри d) следствие вправо. это очень просто следует напрямую. в e) рассматриваются [math]A \setminus B[/math] и [math]B[/math] - т. е. два непересекающихся множества. из d) следует, что также не пересекаются их образы [math]f(A \setminus B[/math] и [math]f(B)[/math]. значит, [math]f(A \setminus B)[/math] полностью содержится в [math]f(A) \setminus f(B)[/math] как подмножество. именно это [math]f(A \setminus B) \subseteq f(A) \setminus f(B)[/math] нетривиально и специфично для инъекции. для равенства [math]f(A \setminus B) = f(A) \setminus f(B)[/math] еще нужно включение правого в левое: [math]f(A) \setminus f(B) \subseteq f(A \setminus B)[/math], но оно тривиально и неспецифично, и выполняется всегда без дополнительных условий. так из d) следует e).

>из e) следует a)


инъективность: из равенства образов следует равенство прообразов. а равенство элементов - это то же, что невозможность разделить их по непересекающимся множествам. в свою очередь, различие - это возможность такого разделения. для любых множеств [math]A[/math] и [math]B[/math] можно рассмотреть [math]A[/math] как объединение [math]A = (A \cap B) \cup (A \setminus B)[/math] непересекающихся [math]A \cap B[/math] и [math]A \setminus B[/math]. так можно произвольно выбирать множества и пытаться отделить одни элементы от других, тем самым проверяя равны ли эти элементы или различны.
теперь приведем доказательство a) из e). в нем будет много технической писанины, однако главное - ухватить именно изложенную выше суть.
пусть [math]f(x) = f(y)[/math]. тогда рассмотрим [math]A = \{ x \}[/math] и [math]B = \{ y \}[/math]. по условию выходит [math]f(A) = f(B)[/math], значит, [math]f(A) \setminus f(B) = \varnothing[/math]. тогда также [math]f(A \setminus B) = \varnothing[/math], откуда [math]\{ x \} \ subseteq \{ y \}[/math], следовательно [math]x = y[/math]. поскольку из [math]f(x) = f(y)[/math] получили [math]x = y[/math], то [math]f[/math] инъекция по определению.
59 115982
>>5968

>d)


внутри самого d) следствие влево вытекает просто из определения отображения как однозначного отношения. и только следствие вправо нетривиально и связано с инъективностью.
инъективность [math]f[/math] означает, что одинаковые образы обязательно имеют одинаковые прообразы, и никогда не разные: [math]fx = fy \to x=y[/math]. или, иначе говоря, от противного: разные прообразы обязательно имеют разные образы: [math]x \neq y \to fx \neq fy[/math]
следствие вправо говорит, что если образы множеств неких прообразов не имеют общих образов, то сами множества этих прообразов не имеют общих прообразов. как это связано с инъективностью? говорить о различии [math]x[/math] и [math]y[/math]: [math]x \neq y[/math] - это то же самое, что говорить о них как об элементах неких непересекающихся множеств [math]A[/math] и [math]B[/math]. т. о. следствие вправо просто говорит о том, что разные прообразы имеют разные образы, но говорит это с помощью непересекающихся множеств.
конечно, нотация несимпатичная, смешивающая с одной стороны образы и прообразы, и с другой стороны множества образов и прообразов (которые, вообще говоря, могут сами быть среди образов и прообразов того же отображения). надеюсь, нотация нас не запутывает, и хотя я в общем против ее применения, но здесь ее использую.
далее, каким образом

>из d) следует e)


, имея в виду именно содержащееся внутри d) следствие вправо. это очень просто следует напрямую. в e) рассматриваются [math]A \setminus B[/math] и [math]B[/math] - т. е. два непересекающихся множества. из d) следует, что также не пересекаются их образы [math]f(A \setminus B[/math] и [math]f(B)[/math]. значит, [math]f(A \setminus B)[/math] полностью содержится в [math]f(A) \setminus f(B)[/math] как подмножество. именно это [math]f(A \setminus B) \subseteq f(A) \setminus f(B)[/math] нетривиально и специфично для инъекции. для равенства [math]f(A \setminus B) = f(A) \setminus f(B)[/math] еще нужно включение правого в левое: [math]f(A) \setminus f(B) \subseteq f(A \setminus B)[/math], но оно тривиально и неспецифично, и выполняется всегда без дополнительных условий. так из d) следует e).

>из e) следует a)


инъективность: из равенства образов следует равенство прообразов. а равенство элементов - это то же, что невозможность разделить их по непересекающимся множествам. в свою очередь, различие - это возможность такого разделения. для любых множеств [math]A[/math] и [math]B[/math] можно рассмотреть [math]A[/math] как объединение [math]A = (A \cap B) \cup (A \setminus B)[/math] непересекающихся [math]A \cap B[/math] и [math]A \setminus B[/math]. так можно произвольно выбирать множества и пытаться отделить одни элементы от других, тем самым проверяя равны ли эти элементы или различны.
теперь приведем доказательство a) из e). в нем будет много технической писанины, однако главное - ухватить именно изложенную выше суть.
пусть [math]f(x) = f(y)[/math]. тогда рассмотрим [math]A = \{ x \}[/math] и [math]B = \{ y \}[/math]. по условию выходит [math]f(A) = f(B)[/math], значит, [math]f(A) \setminus f(B) = \varnothing[/math]. тогда также [math]f(A \setminus B) = \varnothing[/math], откуда [math]\{ x \} \ subseteq \{ y \}[/math], следовательно [math]x = y[/math]. поскольку из [math]f(x) = f(y)[/math] получили [math]x = y[/math], то [math]f[/math] инъекция по определению.
60 115983
>>5982

>тогда также [math]f(A \setminus B) = \varnothing[/math], откуда [math]\{ x \} subseteq \{ y \}[/math]


[math]\{ x \} \subseteq \{ y \}[/math]
61 115984
>>5976

>3b1b же существует, почему бы всем не равняться на них?


какая аудитория у учебника и какая у ютуб-канала. какие предъявляются требования к достижению результатов в каждом из случаев. это не одно и то же.
image.png110 Кб, 898x972
62 115985
что ей ответить?
image.png131 Кб, 963x963
63 115986
ладно, я переоценил её
64 115987
>>5985
она правильно ответила
65 115988
>>5981
Соглы, ролик про индукцию был лучше
66 115989
>>5985

> спрашивать про математику у ии, так еще и у женщины

67 115990
>>5977

>то, что $ f(A) \setminus f(B) \subset f(A \setminus B) $ это включение для любых множеств верно?


это всегда верно

> как всё-таки доказать, что $ C \setminus D \subset f(A) \cap f(X \setminus A) $.


выше дано правильное рассуждение:
из $B \subset A$ получаем, что $f(A\setminus B) \cap B = \emptyset$
с учётом $f(A\setminus B) \subset f(A)$ отсюда следует $f(A\setminus B) \subset f(A) \setminus f(B)$
68 115991
>>5968
Откуда задача?
69 115992
>>5980
крутое видео, узнал кое-что новое, но под конец загрузился. автор очень умный, неудивительно, что он занимается обучением школьников.
70 115994
>>5991
из зорича
image.png34 Кб, 1224x531
71 115995
Очень нравится подборка задач, но они, к сожалению, без решений и подсказок. Есть что-то похожее, но с решениями?
Screenshot12.png226 Кб, 1213x918
72 115996
Люди, есть вопрос по доказательству, это тоже из Зорича. Почему множество целых чисел $ m : x < q^m $ ограничено снизу?
2024-06-26182257.jpg37 Кб, 1084x93
73 115999
>>5996
Это следует буквально из предыдущего предложения в доказательстве (пикрил). Если бы оно не было ограничено снизу, то какое бы сколь угодно большое число \[N\] мы не взяли, \[1/q^{N}=q^{-N}\] было бы все еще больше, чем \[x\].
image.png178 Кб, 1221x800
74 116003
А как мы получили неравенства $ 3) $ ? Как мы получили $ 2) $ ясно, $\alpha_p q^p \leq x < (\alpha_p + 1)q^p $ это из принципа Архимеда следует, вопросов нет. Но вот откуда в $ 3) $ взялось слагаемое $ \alpha_pq^p $ ? Там сказано, что с использованием 2) и принципа архимеда они это получили, но из принципа архимеда вытекает лишь, что $\alpha_{p-1}q^{p-1} \leq x < (\alpha_{p-1} + 1)q^{p-1}.$
75 116009
Кирилл тюльков если ты в этом треде саси хуй
учебник математики себе в очко засунь
76 116011
>>6009

>


твой профессор?
77 116017
Посоветуйте какой-нибудь задачник, можно зарубежный, где хорошая подборка задач по типу тех, что спрашиваются на олимпиадах для 6-7-классников (мат. праздник, например) и на дурацких собесах во всякие йоба-компании.
То есть, игры, конструкции, процессы, алгоритмы, взвешивания, раскраски, текстовые задачи, рыцари-лжецы, кузнечики всякие ебаные, которые прыгают по окружности, математические головоломки и т.д.
смаллиан0001.jpg579 Кб, 1200x2070
78 116018
image.png34 Кб, 1128x116
79 116019
Как строго доказать пункт б)? В этом видосе https://www.youtube.com/watch?v=OtNwlDO2Tj8 на 4:20:33 начинается доказательство, но в конце он плохо рассказал, непонятно и не строго.
80 116020
>>6017
Ну эти, ленинградские кружки
81 116021
>>6019

>


>Как строго доказать пункт б)?


по индукции
82 116022
>>6021
Мне кажется, там и другие аксиомы Пеано придётся использовать
image.png4 Кб, 461x60
83 116023
Как доказать без Бинома? Для четных n. Для нечетных все понятно - можно и заметить, что все посокращается, и взаимно-однозначное соответствие построить между четными и нечетными подмножествами множества из n элементов.
84 116024
Хотел понять, что такое производная, начал видео всякие смотреть и в итоге еще сильнее запутался.
lim ∆X ->0 (f(Xo+∆X) - f(Xo)) / (∆X ) - это определение производной, как я понял. Так почему, если ∆X у нас стремится к нулю, просто тогда не записывают производную так f(Xo)/(Xo)? Я не понимаю.
85 116025
>>6023
Это следует из того, что количество подмножеств у конечного множества с чётным числом элементов совпадает с количеством подмножеств с нечётным числом элементов.

Пусть имеется конечное множество $ X $ с чётным числом элементов. Возьмём какой-либо элемент этого множества $a \in X $. Построим взаимно однозначное соответствие (между подмножествами с чётным и нечётным количеством элементов) следующим образом рассмотрим произвольное подмножество $A \subset X \setminus \{a\} $ и рассмотрим $A \cup \{a\} $ в одном из этих множеств чётное количество элементов, а в другом нечётное. Поставим в соответствие чётному множеству нечётное, получим взаимнооднозначное соответствие.

___________
витя
86 116028
>>6023
Без бинома Ньютона можно, например, так доказать. Возьми абелево многообразие X размерности n над C, рассмотри отображение сдвига x->x+a для некоторого фиксированного a из X. Затем напиши формулу следа Лефшеца для этого отображения. С одной стороны у тебя будет 0, поскольку у этого отображения нет неподвижных точек, а с другой стороны у тебя будет альтернированная сумма следов оператора сдвига на a на всех когомологиях. Затем заметь, что отображение сдвига непрерывно зависит от a, поэтому ты можешь взять a=0 и получить, что альтернированная сумма чисел Бетти с одной стороны равна опять же нулю с другой стороны (потому что след тождественного оператора как раз и дает размерность твоего векторного пространства). Ну и наконец нужно вспомнить как считаются когомологии абелевых многообразий у структурного пучка, просто будут внешние степени пространства антилинейных форм (само пространство антилинейных форм будет размерности как раз n, а его внешние степени размерности C(n,k)). Поэтому числа Бетти у тебя будут как раз твои биномиальные коэффициенты. Ну и в итоге ты получишь что хотел.
87 116029
Как же ваша математика наёбка, пиздец просто

x = - 4
x ^ 2 = (-4) ^ 2
x ^ 2 = 16
x = square root of 16
x1 = 4, x2 = -4
88 116030
>>6019

>В этом видосе https://www.youtube.com/watch?v=OtNwlDO2Tj8[РАСКРЫТЬ] на 4:20:33 начинается доказательство, но в конце он плохо рассказал, непонятно и не строго.


стример не аккуратен. он допустил ошибку. ошибка не принципиальная, но может сбить тебя с толку.
89 116031
>>6022

>Мне кажется, там и другие аксиомы Пеано придётся использовать


я не знаю в точности, что в этой книге обозначается как аксиомы Пеано. но на мой взгляд логично начать рассуждение с [math]x \leq 0 \ x = 0[/math].
90 116032
>>6031

>[math]x \leq 0 \ x = 0[/math]


[math]x \leq 0 \to x = 0[/math]
91 116033
>>6029
хз брат че ты так категоричен, вот есть дорога (-4) и ты благодаря ей пришел к магазину (16), скупился, выходишь и выдишь перед собой две дороги (-4 и 4), которые вели к магазину
92 116038
>>6029
это называется неравносильный переход, у вас в 7 классе тема такая тема появится.

то есть после определённого шага, ты уже не можешь с достоверностью, определить состояние системы на прошлом шаге, если не сделаешь себе никаких дополнительных пометок, т. е. происходит "затирание информации". Остаются только варианты, либо было вот так, либо было так. ты возвёл уравнение в квадрат, но нигде не дописал, что икс был больше нуля. из за этого получил ещё один корень. Математика, как раз таки и занимается тем, что отделяет равносильные переходы от неравносильных.
93 116039
Пик 1 задача. Пик 2 решение. Я тупой или тут какое-то наебалово? Почему в условии говорится, что если за взвешивание отдаешь фальшивую монету, то получишь рандомный результат, а в ответе, что результат не зависит от того, какая монета досталась взвешивателю? И вообще, в решении факт рандома никак не фигурирует.
94 116040
>>6039
Ок, согласен, понял что это я затупил.
95 116041
>>6038
Спасибо за такой подробный ответ.

> ты возвёл уравнение в квадрат, но нигде не дописал, что икс был больше нуля


Но на втором шаге я возвёл в квадрат, но x как раз-таки меньше нуля

>>6033
Спасибо
96 116042
Не подскажите, пожалуйста, почему для равномерно заряженной плоскости E=σ/(2ε0), откуда там двойка?
97 116043
>>6042
У плоскости 2 стороны.
98 116044
Посоветуйте аниме про математику
99 116045
>>6044
оооо я давно такое искал, и на дваче спрашивал, но ничего подобного не нашол. предположу, что японцев настолько сильно мучают школьными предметами, что они даже их упоминания боятся.
100 116046
>>6041

> Но на втором шаге я возвёл в квадрат, но x как раз-таки меньше нуля


да да просто у меня память 6 секунд, естественно х меньше нуля
101 116047
>>6019
докажем по индукции, будем для натурального числа [math]k[/math] рассматривать утверждение о конечности множества [math]E_k[/math] как о неравномощности никакому собственному подмножеству
рассмотрим [math]E_0[/math]. по определению [math]E_0 = \{ x \in N_0 \colon x \leq 0 \}[/math], и поскольку для любого [math]x[/math] из [math]N_0[/math] [math]x \leq 0 \leftrightarrow x = 0[/math], значит, [math]E_0 = \{ 0 \}[/math]
пусть [math]S[/math] - собственное подмножество [math]E_0[/math]. тогда по определению существует такое [math]x[/math], что [math]x \in {/ 0 /}[/math] и [math]x \notin S[/math], откуда [math]0 \notin S[/math], и тогда для любого [math]x[/math] [math]x \in E_0 \to x \notin S[/math]. также для любого [math]x[/math] имеем тавтологию [math]x \in E_0 \lor x \notin E_0[/math]. поскольку теперь как из первого, так и из второго члена следует [math]x \notin S[/math], то [math]S = \varnothing[/math]
если [math]W[/math] - отношение между [math]E_0[/math] и [math]S[/math], то [math]D(W) = \varnothing[/math], откуда [math]E_0 \nsubseteq D(W)[/math] и тогда [math]W[/math] не удовлетворяет определению и не является биекцией между [math]E_0[/math] и [math]S[/math]. поскольку [math]E_0[/math] не равномощно никакому собственному подмножеству [math]S[/math], то [math]E_0[/math] конечно
пусть для некоторого натурального числа [math]k E_k конечно. рассмотрим [math]E_{k+1}[/math]. чтобы доказать конечность [math]E_{k+1}[/math], нам следует показать, что для любой биекции [math]f[/math] между ним и его подмножеством [math]R(f)[/math] не является собственным и равно [math]E_{k+1}[/math]. это можно будет сделать, поскольку [math]E_k[/math] и [math]E_{k+1}[/math] отличаются только на один элемент и, значит, действующие на них отображения тесно связаны между собой. благодаря этому при рассмотрении последнего мы можем пользоваться предположениями о первом
итак, рассмотрим инъекцию [math]f[/math], действующую на [math]E_{k+1}[/math]. теперь определим такую биекцию на [math]E_{k+1}[/math], которая будет иметь такую же область значений [math]R(f)[/math], при этом переведет k+1 в k+1, благодаря чему ее сужение на [math]E_k[/math] будет замкнуто на [math]E_k[/math] и, значит, в силу сохранения инъективности сужение будет иметь область определения равную [math]E_k[/math] по предположению, а тогда область определения самого отображения будет [math]E_{k+1}[/math]
для определения биекции мы возьмем само [math]f[/math] и подправим ту его часть, которая могла бы нарушать замкнутость на [math]E_k[/math], а именно множество пар [math]\{(k+1, f(k+1)), (f^{-1}(k+1), k+1)\}[/math]. определить биекцию можно как множество [math](f \setminus \{(k+1, f(k+1)), (f^{-1}(k+1), k+1)\}) \cup \{ (k+1, k+1), (f^{-1}(k+1), f(k+1)) \}[/math], либо через сужение ли, аксиому ли выделения или композицию - в зависимости от того, в каком случае считается проще доказать, что данное множество является отношением, отображением и биекцией. я предпочитаю первый вариант, доказательство получается простое. в конце концов, поскольку у нас биекция, замкнутая на [math]E_k[/math], то область значений сужения не является несобственным подмножеством и равна [math]E_k[/math], а область значений биекции равна [math]E_{k+1}[/math], откуда [math]R(f) = E_{k+1}[/math]. тогда [math]E_{k+1}[/math] не равномощно никакому собственному подмножеству и является конечным. ч. т. д.
101 116047
>>6019
докажем по индукции, будем для натурального числа [math]k[/math] рассматривать утверждение о конечности множества [math]E_k[/math] как о неравномощности никакому собственному подмножеству
рассмотрим [math]E_0[/math]. по определению [math]E_0 = \{ x \in N_0 \colon x \leq 0 \}[/math], и поскольку для любого [math]x[/math] из [math]N_0[/math] [math]x \leq 0 \leftrightarrow x = 0[/math], значит, [math]E_0 = \{ 0 \}[/math]
пусть [math]S[/math] - собственное подмножество [math]E_0[/math]. тогда по определению существует такое [math]x[/math], что [math]x \in {/ 0 /}[/math] и [math]x \notin S[/math], откуда [math]0 \notin S[/math], и тогда для любого [math]x[/math] [math]x \in E_0 \to x \notin S[/math]. также для любого [math]x[/math] имеем тавтологию [math]x \in E_0 \lor x \notin E_0[/math]. поскольку теперь как из первого, так и из второго члена следует [math]x \notin S[/math], то [math]S = \varnothing[/math]
если [math]W[/math] - отношение между [math]E_0[/math] и [math]S[/math], то [math]D(W) = \varnothing[/math], откуда [math]E_0 \nsubseteq D(W)[/math] и тогда [math]W[/math] не удовлетворяет определению и не является биекцией между [math]E_0[/math] и [math]S[/math]. поскольку [math]E_0[/math] не равномощно никакому собственному подмножеству [math]S[/math], то [math]E_0[/math] конечно
пусть для некоторого натурального числа [math]k E_k конечно. рассмотрим [math]E_{k+1}[/math]. чтобы доказать конечность [math]E_{k+1}[/math], нам следует показать, что для любой биекции [math]f[/math] между ним и его подмножеством [math]R(f)[/math] не является собственным и равно [math]E_{k+1}[/math]. это можно будет сделать, поскольку [math]E_k[/math] и [math]E_{k+1}[/math] отличаются только на один элемент и, значит, действующие на них отображения тесно связаны между собой. благодаря этому при рассмотрении последнего мы можем пользоваться предположениями о первом
итак, рассмотрим инъекцию [math]f[/math], действующую на [math]E_{k+1}[/math]. теперь определим такую биекцию на [math]E_{k+1}[/math], которая будет иметь такую же область значений [math]R(f)[/math], при этом переведет k+1 в k+1, благодаря чему ее сужение на [math]E_k[/math] будет замкнуто на [math]E_k[/math] и, значит, в силу сохранения инъективности сужение будет иметь область определения равную [math]E_k[/math] по предположению, а тогда область определения самого отображения будет [math]E_{k+1}[/math]
для определения биекции мы возьмем само [math]f[/math] и подправим ту его часть, которая могла бы нарушать замкнутость на [math]E_k[/math], а именно множество пар [math]\{(k+1, f(k+1)), (f^{-1}(k+1), k+1)\}[/math]. определить биекцию можно как множество [math](f \setminus \{(k+1, f(k+1)), (f^{-1}(k+1), k+1)\}) \cup \{ (k+1, k+1), (f^{-1}(k+1), f(k+1)) \}[/math], либо через сужение ли, аксиому ли выделения или композицию - в зависимости от того, в каком случае считается проще доказать, что данное множество является отношением, отображением и биекцией. я предпочитаю первый вариант, доказательство получается простое. в конце концов, поскольку у нас биекция, замкнутая на [math]E_k[/math], то область значений сужения не является несобственным подмножеством и равна [math]E_k[/math], а область значений биекции равна [math]E_{k+1}[/math], откуда [math]R(f) = E_{k+1}[/math]. тогда [math]E_{k+1}[/math] не равномощно никакому собственному подмножеству и является конечным. ч. т. д.
102 116048
>>6047

>тогда по определению существует такое [math]x[/math], что [math]x \in {/ 0 /}[/math]


[math]x \in /{ 0 /}[/math]

>откуда [math]E_0 \nsubseteq D(W)[/math]


[math]E_0 \neq D(W)[/math]

>поскольку [math]E_0[/math] не равномощно никакому собственному подмножеству [math]S[/math]


[math]E_0[/math] не равномощно никакому своему собственному подмножеству [math]S[/math]
103 116049
>>6047
>>6048

>[math]x \in /{ 0 /}[/math]


[math]x \in \{ 0 \}[/math]

>откуда [math]E_0 \nsubseteq D(W)[/math]


[math]E_0 \neq D(W)[/math]

>поскольку [math]E_0[/math] не равномощно никакому собственному подмножеству [math]S[/math]


поскольку [math]E_0[/math] не равномощно никакому своему собственному подмножеству [math]S[/math]
104 116050
>>6044
>>6045
Есть серия образовательная манга. Но на самом деле это не так весело, как кажется по обложке, наоборот, ощущение, что все сделано как-то максимально тупо и уныло. Бессмысленный, никуда не двигающийся, картонный сюжет: о привет, ты тоже записался в этот кружок, ну да у меня родители уехали в Киото, так что мне все равно нечего делать по вечерам... мммм я люблю бомжовку со вкусом курицы, а ты какую бомжовку любишь... вау, теперь, когда мы ходим в этот кружок мы стали гораздо умнее, ну ладно, пока, хихихи. Плюс никак не связанные с сюжетом вставки учебного материала на уровне средненей научпоп книжки для школьников, почем учебная часть не только никак не взаимодействует с сюжетной, но и построена как-то странно, когда после примера для детского сада с человечеами, тарелками и креветками могут въебать неизвестно откуда появившийся интеграл. В общем, такое себе, с какой стороны не глянь.
105 116051
>>6050
Да да, про это серию только ленивый не слышал, художественная часть там только для галочки
106 116052
>>6050

>странно, когда после примера для детского сада с человечеами, тарелками и креветками могут [всунуть] неизвестно откуда появившийся интеграл


разве ты не изучал интегралы в детском саду
107 116053
>>6051
Такое ощущение, что там вообще все для галочки. По уму вместо бессмысленного сюжета нужно было бы заполнить тот же объем большим количеством различных примеров, какими-нибудь историческими фактами, как и почему возникла необходимость решения той или иной задачи, забавными фактами по теме. Оформить это в стиле манги, но только чтобы большая часть рисунков несла смысловую нагрузку. Вот это был бы реально топчик. Но тут видимо самим авторам не хватило объема знаний, поэтому получилась компиляция графомании со статьями из вики.
108 116054
>>6046
Ну я понял теперь, после твоего объяснения. Спасибо, анончик, добра!
109 116055
>>6050
Читал, такое же ощущение. После этой серии перестал верить в возможность саморазвития через анимэ. Учёба - это труд.

А любая книга или курс, не предусматривающие что ты самостоятельно решаешь задания хотя бы 50% времени - это шляпа полная
110 116056
Л
111 116057
>>6024
привет

> Хотел понять, что такое производная, начал видео всякие смотреть и в итоге еще сильнее запутался.


понимаю, бывает

>lim ∆X ->0 (f(Xo+∆X) - f(Xo)) / (∆X ) - это определение производной, как я понял.


так и есть, грубо говоря

>Так почему, если ∆X у нас стремится к нулю, просто тогда не записывают производную так f(Xo)/(Xo)? Я не понимаю.


как же можно так просто переписать? ведь вы видите, что даже опустив предел, будут совсем разные выражения:
$\frac{f(x_0 + \Delta x) - f(x_0)}{\Delta x}$
и
$\frac{f(x_0)}{x_0}$.
в знаменателе первого стоит $\Delta x$, а в знаменателе последнего стоит $x_0$. обращается внимание на приращение $\Delta x$ аргумента, а не на значение $x_0$. по своему смыслу эти величины совершенно различные.
так же и в числителе первого стоит приращение функции ${f(x_0 + \Delta x) - f(x_0)}$, а не ее значение $f(x_0)$. и эти величины совершенно различны по смыслу.
может быть, хотя различие видно глазами, но мысль толкает к смешению того и другого? в таком случае, вероятно, было непонято само понятие производной и та мотивация, которая ведет к его рассмотрению. в Википедии дано (https://ru.wikipedia.org/wiki/Производная_функции ) достаточно полезное объяснение:

>При описании процессов и в теории управления производную рассматривают как реакцию процесса (функции) на управляющий этим процессом параметр (независимое переменное); насколько интенсивно реагирует процесс на управляющий сигнал (насколько он чувствителен к нему); какое изменение процесса вызывает небольшое изменение управляющего воздействия.


пусть процесс описывается функцией $f$. насколько чувствительна величина $f(x)$ к изменению вводного $x$? чтобы характеризовать чувствительность количественно, разумно рассмотреть отношение $\frac{\text{изменение зависимой величины}}{\text{изменение вводной величины}}$. поскольку нас интересует изменение относительно конкретного $x_0$, то обычно мы должны брать достаточно малое изменение $\Delta x$. когда мы нуждаемся в математической строгости, то рассматриваем предел $\lim _{\Delta x \to 0} \frac{f(x_0 + \Delta x) - f(x_0)}{\Delta x}$. тут нужно знать понятие предела.
111 116057
>>6024
привет

> Хотел понять, что такое производная, начал видео всякие смотреть и в итоге еще сильнее запутался.


понимаю, бывает

>lim ∆X ->0 (f(Xo+∆X) - f(Xo)) / (∆X ) - это определение производной, как я понял.


так и есть, грубо говоря

>Так почему, если ∆X у нас стремится к нулю, просто тогда не записывают производную так f(Xo)/(Xo)? Я не понимаю.


как же можно так просто переписать? ведь вы видите, что даже опустив предел, будут совсем разные выражения:
$\frac{f(x_0 + \Delta x) - f(x_0)}{\Delta x}$
и
$\frac{f(x_0)}{x_0}$.
в знаменателе первого стоит $\Delta x$, а в знаменателе последнего стоит $x_0$. обращается внимание на приращение $\Delta x$ аргумента, а не на значение $x_0$. по своему смыслу эти величины совершенно различные.
так же и в числителе первого стоит приращение функции ${f(x_0 + \Delta x) - f(x_0)}$, а не ее значение $f(x_0)$. и эти величины совершенно различны по смыслу.
может быть, хотя различие видно глазами, но мысль толкает к смешению того и другого? в таком случае, вероятно, было непонято само понятие производной и та мотивация, которая ведет к его рассмотрению. в Википедии дано (https://ru.wikipedia.org/wiki/Производная_функции ) достаточно полезное объяснение:

>При описании процессов и в теории управления производную рассматривают как реакцию процесса (функции) на управляющий этим процессом параметр (независимое переменное); насколько интенсивно реагирует процесс на управляющий сигнал (насколько он чувствителен к нему); какое изменение процесса вызывает небольшое изменение управляющего воздействия.


пусть процесс описывается функцией $f$. насколько чувствительна величина $f(x)$ к изменению вводного $x$? чтобы характеризовать чувствительность количественно, разумно рассмотреть отношение $\frac{\text{изменение зависимой величины}}{\text{изменение вводной величины}}$. поскольку нас интересует изменение относительно конкретного $x_0$, то обычно мы должны брать достаточно малое изменение $\Delta x$. когда мы нуждаемся в математической строгости, то рассматриваем предел $\lim _{\Delta x \to 0} \frac{f(x_0 + \Delta x) - f(x_0)}{\Delta x}$. тут нужно знать понятие предела.
112 116061
>>6024
Думай
114 116067
Есть ли какие-нибудь няшные книги по алгебре, топологии и анализу для вчерашних школьников, похожие по стилю на книгу "Начала теории множеств" у Шеня? С решениями, комментариями и интересными задачами. Не знаю, куда двигаться после неё, хочу изучить школьную программу по Вербицкому.
115 116068
>>6067
"элементарная топология" Виро и друзей

книжка немного специфическая, но всяко няшная и полезная
116 116069
>>6068
Всратая же книга для вкатуна. Пробовал читать в школе. Выпал с определения топологии, что-то прорешал дальше и забил. Хотя там дальше, кстати, оказалось, что есть слова о метрике и всём таком, но это следовало поместить в начало, как сделал Вербицкий уже в своей книге. Лучше анализ подучить и потом за топологию браться, думаю, хотя бы уметь доказать непрерывность не совсем тривиальных функций.
>>6067
Мне Пинтер по алгебре нравился, он довольно короткий, понятный и есть задачи, а так же ответы, не ко всем, но ко многим задачам.
Потом сможешь взять что-то посерьезней.
По линалу лучше Бурбаков или Булдырева-Павлова ничего наверное нет.
Анализ дефолтный Зорич. Только он почему-то не определяет нормально R, не строит из Q а определяет аксиоматически, хотя у того же Фихнтенгольца есть сечения Дедекинда.
117 116070
>>6067
Наглядная топология Болтянского мб?
118 116071
>>6069

>Всратая же книга для вкатуна


критика адекватная, но не столь уж категоритичная в том смысле, чтобы отменить достоинства этой книги. книга очень хорошая. а потянет её конкретный вкатун или нет, вопрос уже праздный
119 116072
>>6067

>"Начала теории множеств" у Шеня


И чего там в ней няшного, литерали совковое говно из жопы.
120 116073
>>6067
обрати внимание на энциклопедию элементарной математики
>>6069

>Только он почему-то не определяет нормально R, не строит из Q а определяет аксиоматически, хотя у того же Фихнтенгольца есть сечения Дедекинда.


вроде бы в Зориче для этих целей используются грани множеств
121 116075
>>6073

>вроде бы в Зориче для этих целей используются грани множеств


Грани это следствие R. Он просто берет за аксиому их существование. Эту аксиому можно доказать, если построить R из Q. Способов не один и не два.
122 116077
Кстати, то, что $ \mathbb{N} = \{ 1, 1+1, 1+1+1, \ldots \}$ это не очевидно, это следует из определения $ \mathbb{N} $ (Как пересечения всех индуктивных множеств содержащих 1) и аксиомы индукции. А из такого вида и свойств аксиом порядка для $ \mathbb{R}$ можно по индукции доказать, что $ 1 \leq n \ \forall n \in \mathbb{N} .$
Кстати, а как избавиться от такой фигни, что я про всякую используемую вещь пытаюсь понять, как она следует из аксиом и из того, что мы уже доказали. Пытаюсь разбирать каждое предложение, каждое слово, и соответственно затрачиваю много времени на это. И при этом не чувствую удовлетворённости, поскольку кажется, что всё-таки не все вещи строго и формально объяснены. Да вот даже в том же Зориче, он дал определение натуральных чисел, да, оно понятное, и соотносится с тем, что было. Но вот после сразу идёт утверждение о том, что сумма и произведение нат. чисел есть нат. число. Откуда в этом множестве взялась сумма? Что значит выражение $n+m $ , как придать ему смысл? Он ни слова про это не пишет. Грустно.
123 116078
>>6077

>Что значит выражение $n+m$, как придать ему смысл? Он ни слова про это не пишет.


в каком-нибудь другом учебнике, наверно, написано (через аксиомы Пеано)
но тратиться слишком много на определение натуральных/вещественных чисел, не стоит, я думаю. чтобы не превратиться в $N$-петуха
image.png114 Кб, 1687x330
124 116079
>>6047
Сори, твоё доказательство мне не очень понятно, поскольку много синтаксических ошибок в нём содержится. Вот, посмотри моё, верно ли тут всё у меня? Народы, проверьте пожалуйста. Если что, доказываю 4. б) на пикриле.

Докажем утверждение по индукции. $E_n = \{x \in \mathbb{N} \mid x \leq n\}.$

База: $n = 1.$ $E_1 = \{x \mid x \leq 1\}.$ Значит $E_1 = \{1\}$ и у него одно собственное подмножество $\varnothing, $ и оно ему ясное дело не равномощно.

Переход.

Пусть для любого $ i < k+1$ верно, что множество $E_i$ конечно. Рассмотрим $E_{k+1}.$ От противного, пусть $\exists \ E^ \varsubsetneq
E_{k+1} \colon \exists \ \varphi \colon E_{k+1} \to E^
$ -- биекция. Тогда $\varphi \colon \underbrace{ E_{k+1} \setminus \{k+1\} }_{=E_k} \to E^ \setminus \{\varphi(k+1)\}$ -- тоже биекция. Возможны 2 случая.

1) Когда $k+1 \notin E^
\setminus \{\varphi(k+1)\} .$ А это значит, что $E^ \setminus \{\varphi(k+1)\} \varsubsetneq E_k$ -- противоречит предположению индукции. Последнее следует из того, что $E^ \varsubsetneq E_{k+1} ,$ ведь если бы мы имели $E^ \setminus \{\varphi(k+1)\} = E_k$ то из этого бы следовало, что $\left(E^ \setminus \{\varphi(k+1)\}\right) \cup \{k+1\} = E_k \cup \{k+1\} = E_{k+1}$ -- противоречие.

2) Когда $k+1 \in E^ \setminus \{\varphi(k+1)\} .$ Построим биективное отображение $g$ из множества $ E^ \setminus \{\varphi(k+1)\}, $ $g$ будет биекцией на образ. Положим $g(\alpha) = \alpha$ если $\alpha \neq k+1$ и $g(k+1) = \varphi(k+1).$ Проверим инъективность $g.$ Т.к. $\varphi$ -- биекция, то $\exists! \ \gamma \in E_k \colon \gamma = \varphi(k+1)$ и если $g(k+1) = g(\alpha) $, $\alpha \neq k+1,$ то $g(k+1) = \varphi(k+1) = \alpha = \gamma,$ но $\gamma = \varphi(k+1) \notin E^ \setminus \{\varphi(k+1)\}.$ Значит $\gamma \neq \alpha \ \forall \alpha \in E^ \setminus \{\varphi(k+1)\} \Longrightarrow$ $g$ -- инъекция $\Longrightarrow g$ -- биекция на образ $E' = g(E^ \setminus \{\varphi(k+1)\})$. Для завершения доказательства осталось показать, что $E' \varsubsetneq E_k$, а после рассматривая композицию биекций $\varphi \circ g$ получим нужное утверждение. Можно заметить, что $E' = E^ \setminus \{k+1\}, $ а поскольку $E^* \varsubsetneq E_{k+1}$ удаляя из обеих частей этого соотношения по элементу $k+1$ получим необходимое соотношение. ч.т.д.
image.png114 Кб, 1687x330
124 116079
>>6047
Сори, твоё доказательство мне не очень понятно, поскольку много синтаксических ошибок в нём содержится. Вот, посмотри моё, верно ли тут всё у меня? Народы, проверьте пожалуйста. Если что, доказываю 4. б) на пикриле.

Докажем утверждение по индукции. $E_n = \{x \in \mathbb{N} \mid x \leq n\}.$

База: $n = 1.$ $E_1 = \{x \mid x \leq 1\}.$ Значит $E_1 = \{1\}$ и у него одно собственное подмножество $\varnothing, $ и оно ему ясное дело не равномощно.

Переход.

Пусть для любого $ i < k+1$ верно, что множество $E_i$ конечно. Рассмотрим $E_{k+1}.$ От противного, пусть $\exists \ E^ \varsubsetneq
E_{k+1} \colon \exists \ \varphi \colon E_{k+1} \to E^
$ -- биекция. Тогда $\varphi \colon \underbrace{ E_{k+1} \setminus \{k+1\} }_{=E_k} \to E^ \setminus \{\varphi(k+1)\}$ -- тоже биекция. Возможны 2 случая.

1) Когда $k+1 \notin E^
\setminus \{\varphi(k+1)\} .$ А это значит, что $E^ \setminus \{\varphi(k+1)\} \varsubsetneq E_k$ -- противоречит предположению индукции. Последнее следует из того, что $E^ \varsubsetneq E_{k+1} ,$ ведь если бы мы имели $E^ \setminus \{\varphi(k+1)\} = E_k$ то из этого бы следовало, что $\left(E^ \setminus \{\varphi(k+1)\}\right) \cup \{k+1\} = E_k \cup \{k+1\} = E_{k+1}$ -- противоречие.

2) Когда $k+1 \in E^ \setminus \{\varphi(k+1)\} .$ Построим биективное отображение $g$ из множества $ E^ \setminus \{\varphi(k+1)\}, $ $g$ будет биекцией на образ. Положим $g(\alpha) = \alpha$ если $\alpha \neq k+1$ и $g(k+1) = \varphi(k+1).$ Проверим инъективность $g.$ Т.к. $\varphi$ -- биекция, то $\exists! \ \gamma \in E_k \colon \gamma = \varphi(k+1)$ и если $g(k+1) = g(\alpha) $, $\alpha \neq k+1,$ то $g(k+1) = \varphi(k+1) = \alpha = \gamma,$ но $\gamma = \varphi(k+1) \notin E^ \setminus \{\varphi(k+1)\}.$ Значит $\gamma \neq \alpha \ \forall \alpha \in E^ \setminus \{\varphi(k+1)\} \Longrightarrow$ $g$ -- инъекция $\Longrightarrow g$ -- биекция на образ $E' = g(E^ \setminus \{\varphi(k+1)\})$. Для завершения доказательства осталось показать, что $E' \varsubsetneq E_k$, а после рассматривая композицию биекций $\varphi \circ g$ получим нужное утверждение. Можно заметить, что $E' = E^ \setminus \{k+1\}, $ а поскольку $E^* \varsubsetneq E_{k+1}$ удаляя из обеих частей этого соотношения по элементу $k+1$ получим необходимое соотношение. ч.т.д.
125 116080
Докажем утверждение по индукции. $E_n = \{x \in \mathbb{N} \mid x \leq n\}.$

База: $n = 1.$ $E_1 = \{x \mid x \leq 1\}.$ Значит $E_1 = \{1\}$ и у него одно собственное подмножество $\varnothing, $ и оно ему ясное дело не равномощно.

Переход.

Пусть для любого $ i < k+1$ верно, что множество $E_i$ конечно. Рассмотрим $E_{k+1}.$ От противного, пусть $\exists \ E^ \varsubsetneq
E_{k+1} \colon \exists \ \varphi \colon E_{k+1} \to E^
$ -- биекция. Тогда $\varphi \colon \underbrace{ E_{k+1} \setminus \{k+1\} }_{=E_k} \to E^ \setminus \{\varphi(k+1)\}$ -- тоже биекция. Возможны 2 случая.

1) Когда $k+1 \notin E^
\setminus \{\varphi(k+1)\} .$ А это значит, что $E^ \setminus \{\varphi(k+1)\} \varsubsetneq E_k$ -- противоречит предположению индукции. Последнее следует из того, что $E^ \varsubsetneq E_{k+1} ,$ ведь если бы мы имели $E^ \setminus \{\varphi(k+1)\} = E_k$ то из этого бы следовало, что $\left(E^ \setminus \{\varphi(k+1)\}\right) \cup \{k+1\} = E_k \cup \{k+1\} = E_{k+1}$ -- противоречие.

2) Когда $k+1 \in E^ \setminus \{\varphi(k+1)\} .$ Построим биективное отображение $g$ из множества $ E^ \setminus \{\varphi(k+1)\}, $ $g$ будет биекцией на образ. Положим $g(\alpha) = \alpha$ если $\alpha \neq k+1$ и $g(k+1) = \varphi(k+1).$ Проверим инъективность $g.$ Т.к. $\varphi$ -- биекция, то $\exists! \ \gamma \in E_k \colon \gamma = \varphi(k+1)$ и если $g(k+1) = g(\alpha) $, $\alpha \neq k+1,$ то $g(k+1) = \varphi(k+1) = \alpha = \gamma,$ но $\gamma = \varphi(k+1) \notin E^ \setminus \{\varphi(k+1)\}.$ Значит $\gamma \neq \alpha \ \forall \alpha \in E^ \setminus \{\varphi(k+1)\} \Longrightarrow$ $g$ -- инъекция $\Longrightarrow g$ -- биекция на образ $E' = g(E^ \setminus \{\varphi(k+1)\})$. Для завершения доказательства осталось показать, что $E' \varsubsetneq E_k$, а после рассматривая композицию биекций $\varphi \circ g$ получим нужное утверждение. Можно заметить, что $E' = E^ \setminus \{k+1\}, $ а поскольку $E^* \varsubsetneq E_{k+1}$ удаляя из обеих частей этого соотношения по элементу $k+1$ получим необходимое соотношение. ч.т.д.

>>6079
125 116080
Докажем утверждение по индукции. $E_n = \{x \in \mathbb{N} \mid x \leq n\}.$

База: $n = 1.$ $E_1 = \{x \mid x \leq 1\}.$ Значит $E_1 = \{1\}$ и у него одно собственное подмножество $\varnothing, $ и оно ему ясное дело не равномощно.

Переход.

Пусть для любого $ i < k+1$ верно, что множество $E_i$ конечно. Рассмотрим $E_{k+1}.$ От противного, пусть $\exists \ E^ \varsubsetneq
E_{k+1} \colon \exists \ \varphi \colon E_{k+1} \to E^
$ -- биекция. Тогда $\varphi \colon \underbrace{ E_{k+1} \setminus \{k+1\} }_{=E_k} \to E^ \setminus \{\varphi(k+1)\}$ -- тоже биекция. Возможны 2 случая.

1) Когда $k+1 \notin E^
\setminus \{\varphi(k+1)\} .$ А это значит, что $E^ \setminus \{\varphi(k+1)\} \varsubsetneq E_k$ -- противоречит предположению индукции. Последнее следует из того, что $E^ \varsubsetneq E_{k+1} ,$ ведь если бы мы имели $E^ \setminus \{\varphi(k+1)\} = E_k$ то из этого бы следовало, что $\left(E^ \setminus \{\varphi(k+1)\}\right) \cup \{k+1\} = E_k \cup \{k+1\} = E_{k+1}$ -- противоречие.

2) Когда $k+1 \in E^ \setminus \{\varphi(k+1)\} .$ Построим биективное отображение $g$ из множества $ E^ \setminus \{\varphi(k+1)\}, $ $g$ будет биекцией на образ. Положим $g(\alpha) = \alpha$ если $\alpha \neq k+1$ и $g(k+1) = \varphi(k+1).$ Проверим инъективность $g.$ Т.к. $\varphi$ -- биекция, то $\exists! \ \gamma \in E_k \colon \gamma = \varphi(k+1)$ и если $g(k+1) = g(\alpha) $, $\alpha \neq k+1,$ то $g(k+1) = \varphi(k+1) = \alpha = \gamma,$ но $\gamma = \varphi(k+1) \notin E^ \setminus \{\varphi(k+1)\}.$ Значит $\gamma \neq \alpha \ \forall \alpha \in E^ \setminus \{\varphi(k+1)\} \Longrightarrow$ $g$ -- инъекция $\Longrightarrow g$ -- биекция на образ $E' = g(E^ \setminus \{\varphi(k+1)\})$. Для завершения доказательства осталось показать, что $E' \varsubsetneq E_k$, а после рассматривая композицию биекций $\varphi \circ g$ получим нужное утверждение. Можно заметить, что $E' = E^ \setminus \{k+1\}, $ а поскольку $E^* \varsubsetneq E_{k+1}$ удаляя из обеих частей этого соотношения по элементу $k+1$ получим необходимое соотношение. ч.т.д.

>>6079
126 116081
какого чёрта текст так отобразился
127 116082
>>6081
ужасные вещи случаются, когда берёшься доказывать естественно очевидные вещи
128 116083
>>6082
они то естественные и очевидные, но это же не значит, что их не нужно доказывать, или не стоит искать путь их доказательства, верно? профессиональный математик должен уметь доказывать такие вещи.
129 116084
>>6075

>Грани это следствие R. Он просто берет за аксиому их существование. Эту аксиому можно доказать, если построить R из Q. Способов не один и не два.


понимаю
кнч построение модели важно для непротиворечивости
или есть еще какие-то мотивации к построению?
130 116085
>>6083
их не нужно доказывать - зачем? такое доказательство нигде не используется и едва ли улучшает понимание

профессиональный математик совершенно не должен уметь доказывать такие вещи, потому как доказывать их нет никакой надобности - все эти вопросы уже давно закрыты, кроме разве что в самих основаниях, где в каждой системе аксиом требуется 2+2 вычислять заново

наконец, когда речь идёт про такие утверждения, слишком близкие к формальной аксиоматике, становится уже не слишком понятно, что такое вообще доказательство

если опираться на наивную логику и здравый смысл (основной метод в современной математике), "доказательство" этих утверждений превращается в неудобоваримый трэш, который даже записать трудно (что мы и наблюдаем сейчас)
image.png196 Кб, 1140x949
131 116086
>>6085

>превращается в неудобоваримый трэш, который даже записать трудно (что мы и наблюдаем сейчас)



Забавное замечание xdd, вот прилагаю скрин, на котором всё более менее аккуратно.

Просто иногда порой задаёшься вопросом, а вот как формально доказать ту или иную вещь, которая кажется очевидной. Я просто хочу стать профессиональным математиком, вот начал повторять матан заново, и параллельно задаюсь вопросами, которыми раньше не задавался.

Кстати, а в каком университете обучаешься, если не секрет?
132 116087
>>6086
это доказательство не формально

я не обучаюсь в университете
133 116088
>>6077

>я про всякую используемую вещь пытаюсь понять, как она следует из аксиом и из того, что мы уже доказали


замечательно. если это твоя личная потребность, то увлеченно занимаясь ты будешь сильным математиком

>Пытаюсь разбирать каждое предложение, каждое слово, и соответственно затрачиваю много времени на это


кто занимается математикой, тот тратит на нее много времени

>И при этом не чувствую удовлетворённости


когда потребность не осознана, не поставлена ясная цель, то нет и ясного пути к удовлетворению потребности. потребность остается неудовлетворенной

>всё-таки не все вещи строго и формально объяснены


все вещи объяснены, строго выражены и воплощены формально. только нужно иметь источники информации и понимать ее.

>Да вот даже в том же Зориче, он дал определение натуральных чисел, да, оно понятное, и соотносится с тем, что было. Но вот после сразу идёт утверждение о том, что сумма и произведение нат. чисел есть нат. число. Откуда в этом множестве взялась сумма? Что значит выражение n+m , как придать ему смысл? Он ни слова про это не пишет. Грустно.


приятно, что тебе грустно ахах лан, это коры хДДД приятно, что ты это увидел и озаботился. твое замечание уместно и остроумно. учебник Зорича могут ценить и называть понятным и строгим, но таким он является только в контексте вуза, а не в специальном смысле. есть др. источники, какие-то из них стремятся к понятности, какие-то - к строгости, а какие-то воплощают строгость в формализме.

>Кстати, а как избавиться от такой фигни


это скромное хвастовство? не нужно избавляться от стремления к пониманию. но можно находить путь через преграды
в частности, обучение математике отличается от обучения в школе или вузе. одно не должно вредить другому
более глубокой является мировоззренческая проблема. я придерживаюсь морального релятивизма, ценности свободы и плюрализма культур и идей, противостою фундаментализму и одержимости сверхценностями. я считаю изучение математических идей частью культурного обогащения. при этом можно модерировать роль математики в жизни и не позволять полностью ее захватить
изучая математическую идею (к примеру, натуральное число, предел и др.) мы стремимся ее понять. понимая идею, мы стремимся выразить ее строго, т. е. очистить и привести к минимальной основе. благодаря ценности плюрализма, сознанию множественности разнообразных идей, очищение идеи не приводит к ограничению сознания, а только открывает новый вариант. далее строгость можно воплотить в формализме, сведя взаимосвязь к беспристрастному механическому оперированию символами
советую обратить внимание на книгу Феферман Соломон, Числовые системы (http://physics.gov.az/book_Ch/NUMBER_SYSTEMS.pdf ). в ней рассказывается о множествах и в связи с ними о натуральных числах. понимание дается неглубокое, строгость высокая, формализма нет
>>6079

>много синтаксических ошибок в нём содержится


не согласен

>Вот, посмотри моё, верно ли тут всё у меня?


я бы не стал читать такой текст, если бы встретился с малейшим затруднением. по какому-то случаю, я все стал понимать сходу. вопрос был верно ли, поэтому я дочитал до того места, где неверно: $g$ - не инъекция: по условию $k+1 \in \hat E \setminus \{ \varphi (k+1) \}$ и по построению $g(\varphi (k+1)) = \varphi (k+1) = g(k+1)$. наверное, дальше вы рассмотрите сужение, которое будет инъективным, и это можно доказать - это нужно сделать аккуратно, и мысль прослеживается, но что есть, то есть: уже неверно. ту же ошибку отметил в стриме, который упоминался выше.
в целом, ваши доказательства одинаковы. в них есть мысль. в них мне не симпатично форсирование прямого противоречия с данными задачи, отдельно и получившееся из него разбитие на случаи. лично я пришел к доказательству, которое мне нравилось, но к сожалению, я решил сначала посмотреть стрим, а потом его написать, и забыл. в итоге написал новое доказательство, тоже хорошее, но не такое ровное по темпу.
>>6081
юзай https://zohooo.github.io/jaxedit/
>>6086

>вот прилагаю скрин, на котором всё более менее аккуратно


сейчас не буду второй раз читать

>Просто иногда порой задаёшься вопросом, а вот как формально доказать ту или иную вещь, которая кажется очевидной. Я просто хочу стать профессиональным математиком, вот начал повторять матан заново, и параллельно задаюсь вопросами, которыми раньше не задавался.


профессиональная математика и математическая строгость существуют отдельно друг от друга. абсолютное большинство вузовских математиков не выказывают и, по-видимому, не имеют представления о строгости
>>6087

>это доказательство не формально


подтверждаю
133 116088
>>6077

>я про всякую используемую вещь пытаюсь понять, как она следует из аксиом и из того, что мы уже доказали


замечательно. если это твоя личная потребность, то увлеченно занимаясь ты будешь сильным математиком

>Пытаюсь разбирать каждое предложение, каждое слово, и соответственно затрачиваю много времени на это


кто занимается математикой, тот тратит на нее много времени

>И при этом не чувствую удовлетворённости


когда потребность не осознана, не поставлена ясная цель, то нет и ясного пути к удовлетворению потребности. потребность остается неудовлетворенной

>всё-таки не все вещи строго и формально объяснены


все вещи объяснены, строго выражены и воплощены формально. только нужно иметь источники информации и понимать ее.

>Да вот даже в том же Зориче, он дал определение натуральных чисел, да, оно понятное, и соотносится с тем, что было. Но вот после сразу идёт утверждение о том, что сумма и произведение нат. чисел есть нат. число. Откуда в этом множестве взялась сумма? Что значит выражение n+m , как придать ему смысл? Он ни слова про это не пишет. Грустно.


приятно, что тебе грустно ахах лан, это коры хДДД приятно, что ты это увидел и озаботился. твое замечание уместно и остроумно. учебник Зорича могут ценить и называть понятным и строгим, но таким он является только в контексте вуза, а не в специальном смысле. есть др. источники, какие-то из них стремятся к понятности, какие-то - к строгости, а какие-то воплощают строгость в формализме.

>Кстати, а как избавиться от такой фигни


это скромное хвастовство? не нужно избавляться от стремления к пониманию. но можно находить путь через преграды
в частности, обучение математике отличается от обучения в школе или вузе. одно не должно вредить другому
более глубокой является мировоззренческая проблема. я придерживаюсь морального релятивизма, ценности свободы и плюрализма культур и идей, противостою фундаментализму и одержимости сверхценностями. я считаю изучение математических идей частью культурного обогащения. при этом можно модерировать роль математики в жизни и не позволять полностью ее захватить
изучая математическую идею (к примеру, натуральное число, предел и др.) мы стремимся ее понять. понимая идею, мы стремимся выразить ее строго, т. е. очистить и привести к минимальной основе. благодаря ценности плюрализма, сознанию множественности разнообразных идей, очищение идеи не приводит к ограничению сознания, а только открывает новый вариант. далее строгость можно воплотить в формализме, сведя взаимосвязь к беспристрастному механическому оперированию символами
советую обратить внимание на книгу Феферман Соломон, Числовые системы (http://physics.gov.az/book_Ch/NUMBER_SYSTEMS.pdf ). в ней рассказывается о множествах и в связи с ними о натуральных числах. понимание дается неглубокое, строгость высокая, формализма нет
>>6079

>много синтаксических ошибок в нём содержится


не согласен

>Вот, посмотри моё, верно ли тут всё у меня?


я бы не стал читать такой текст, если бы встретился с малейшим затруднением. по какому-то случаю, я все стал понимать сходу. вопрос был верно ли, поэтому я дочитал до того места, где неверно: $g$ - не инъекция: по условию $k+1 \in \hat E \setminus \{ \varphi (k+1) \}$ и по построению $g(\varphi (k+1)) = \varphi (k+1) = g(k+1)$. наверное, дальше вы рассмотрите сужение, которое будет инъективным, и это можно доказать - это нужно сделать аккуратно, и мысль прослеживается, но что есть, то есть: уже неверно. ту же ошибку отметил в стриме, который упоминался выше.
в целом, ваши доказательства одинаковы. в них есть мысль. в них мне не симпатично форсирование прямого противоречия с данными задачи, отдельно и получившееся из него разбитие на случаи. лично я пришел к доказательству, которое мне нравилось, но к сожалению, я решил сначала посмотреть стрим, а потом его написать, и забыл. в итоге написал новое доказательство, тоже хорошее, но не такое ровное по темпу.
>>6081
юзай https://zohooo.github.io/jaxedit/
>>6086

>вот прилагаю скрин, на котором всё более менее аккуратно


сейчас не буду второй раз читать

>Просто иногда порой задаёшься вопросом, а вот как формально доказать ту или иную вещь, которая кажется очевидной. Я просто хочу стать профессиональным математиком, вот начал повторять матан заново, и параллельно задаюсь вопросами, которыми раньше не задавался.


профессиональная математика и математическая строгость существуют отдельно друг от друга. абсолютное большинство вузовских математиков не выказывают и, по-видимому, не имеют представления о строгости
>>6087

>это доказательство не формально


подтверждаю
134 116089
Привет, как человек сурово и очень давно травмированный системой образования, решил самостоятельно пройти школьную математику.
Купил учебник по геометрии за 7-й класс, читаю, и в первой же главе, после изложения того что такое точка, прямая и плоскость, меня поставили в ступор таким вопросом для самопроверки: надо доказать, что 7 прямых могут/не могут попарно пересекаться в
а) в семи точках,
б) в восьми точках.


Как это сделать при помощи знаний о том, что такое точка, прямая и плоскость? Ну, кроме перебора с линейкой, пока не получится.
135 116091
>>6089
Сколько у двух прямых может быть точек пересечения между собой?
136 116093
>>6067

>"Начала теории множеств"


У меня с математикой всё печально тройка по ней в аттестате, но Савватеев вдохновил меня на то, чтобы попытаться её изучить мне понравилась его научно-популярная лекция по топологии с этим множеством красивых бубликов. Книжки для умных ребят с малого мехмата и 57-ой школы у меня ни в какую не идут из-за отсутствия у меня теоретико-множественного мышления, как я думаю. Может быть мне стоит попробовать решить этот задачник? Савватеев говорит, что он хорошо ставит логику и отлично подготавливает к алгебре и матану. Сначала решить его, а уже потом к листочкам и/или Алексееву/Гашкову переходить. Что скажете?
1000012466.jpg155 Кб, 1080x658
137 116094
Прошу помогите решить.
138 116095
Сап
Есть 2 стула задачника. В первом около 120 страниц, теория к каждой главе дается очень кратко и задачи простые. В другом около 600 страниц, более широкий охват тем, но задачи намного сложнее.
Вопрос: какой из задачников мне прорешать? Первый я уже прорешал наполовину. Но знакомый чел предлагает бросить первый задачник и браться за второй, потому что он намного пизже. Я попробовал, но задачки со второго я просто не вывожу. Я пока склоняюсь к тому что дорешать первый задачник и потом переходить к чему-нибудь другому. У меня на самом деле целый список книг и задачников которые я хочу прорешать и если я засяду за второй задачник это мне и лета не хватит на него, а так по моим прикидкам за лето спокойно прорешаю первый задачник и возможно еще что-нибудь.
139 116096
>>6095
делай то, что для тебя работает

главное, не превращайся в петуха-неосилятора, у которого ничего не работает и из-за этого он агрессивно кидается на всё подряд
140 116097
>>6096
Да вот я тоже так думаю, что нужно начинать с простого.
Лучше набить базу на простых задачках, а потом если что можно и вернуться к более сложным.
141 116098
>>6093
Не игнорируйте, пожалуйста, это не пустой вопрос.
142 116099
>>6098
энциклопедия элементарной математики
143 116101
Как понять, что что-то очевидно?
144 116102
>>6093
Скачивай задачник Сканави и ебашь.
145 116103
>>6091
Одна, это-то тут при чём? Они ж про попарные пересечения (термин какой же на слух неприятный) спрашивают.
146 116104
>>6103
на одной и той же плоскости 2 точки определяют прямую
147 116105
>>6104
Как из этого следует что может существовать, ну, допустим, правильный семиугольник/восьмиугольник/n-угольник, например? Там n сторон каждая из которых часть какой-то прямой, т.е. в углах фигуры будет это попарное пересечение. Но как это логически следует из того что существуют точки, прямые и плоскость, и какая разница сколько сторон у этой фигуры будет, зачем вопрос ставится так, что есть разница? Можно не намёками, я очень плохо намёки понимаю.
148 116106
>>6099
Какая-то всратая энциклопедия для чрезмерно одаренных детишек. Задач причём вообще нет, хех.
>>6102
Нет, спасибо, я этого дерьма уже наелся.
149 116107
>>6105

>Можно не намёками, я очень плохо намёки понимаю.


я не вникал в решение задачи, но полагаю, что основная идея такова: прямые могут либо совпадать по 2 точкам, либо пересекаться в 1 точке, либо быть параллельными и тогда иметь 0 общих точек. каким-то образом тебе нужно разрулить для 7 прямых, 7 и 8 точек.
>>6106

>Какая-то всратая энциклопедия для чрезмерно одаренных детишек. Задач причём вообще нет, хех.


это для учителей математики. и что значит: нет задач? изучение материала не уступает задачам, зачем тогда дополнительные задачи?
150 116108
>>6096
Мелкочмошная ебанашка, почему ты утверждаешь что нет ошибки даже после того как тебя как следует потыкали в нее носом?
И как с этой своей непроходимой тупорылостью ты считаешь себя в праве давать какие либо советы хоть кому то в чем то.
151 116110
>>6108
так ну-ка пошел вон отсюда к математике не имеешь отношения все про чмо какое-то пишешь уже сколько месяцев
>>6096
ты тоже вон отсюда пошел постоянно какие-то петухи вы откуда эту грязь тащите свиньи из гулага
152 116111
>>6094
Смог решить все кроме d. Подскажите как этт сделать.
153 116112
>>6106

>У меня с математикой всё печально тройка по ней в аттестате


>попытаться её изучить


есть много разных представлений о математике. стандарт среднего полного образования - наиболее узкий из них. в отдельности от школы он не стоит того, чтобы его изучать

>>"Начала теории множеств"


я бы назвал эту книгу хорошей, если бы за 100 страниц там вводилась аксиоматическая теория множеств. но я не вижу обоснованности такого значительного объема книги при отсутствии в ней аксиоматического подхода

>Может быть мне стоит попробовать решить этот задачник?


>ставит логику и отлично подготавливает к алгебре и матану


>Сначала решить его, а уже потом к листочкам и/или Алексееву/Гашкову переходить


думаю, в твоей ситуации полезно культурно обогатиться различными идеями из математики. здесь нету прямого согласования с углублением в теорию множеств, и тем более с решением уймы задач. если далее ты планируешь изучать алгебру и матан, то ты обнаружишь, что там ты просто будешь узнавать новые идеи, без применения глубоких познаний из теории множеств.
мое расследование связало фамилии Алексеев и Гашков с какими-то книгами по информатике. это иная область, и об этом я решил не размышлять
предлагаю два варианта: энциклопедия элементарной математики или Феферман Соломон, Числовые системы. 100 страниц любой одной из этих книг дает больше для дальнейшего изучения алгебры и матана, чем дает Верещагин, Шень, Начала теории множеств.
154 116114
>>6110
А ты что за кусок говна еще такой? Ебанашка, у тебя раздвоение личности началось уже?
155 116116
>>6112
Я имел ввиду "Теорему Абеля в задачах и решениях" от Алексеева. Это книга по алгебре.
156 116117
>>6093

>Книжки для умных ребят с малого мехмата и 57-ой школы у меня ни в какую не идут


Мне интересно, почему ты решил выбрать именно их? Спойлер - эти книги трата времени, ты все эти темы, которые в них есть, узнаешь из более хороших книг потом, в более приличном виде. Эти книги существует только чтобы надрачивать детей на олимпиады. Если ты не школьник, мечтающий поступить мфти вшэ спбгу мгу, тебе это не нужно.

>Что скажете?


Тебе нужно освоить два приема доказательства. Первый это от противного, а другой индукция.
Возьми Калужнин "основная теорема арифметики". Попытайся решать задачи сам. Если не получается, читай дальше. Там не раз используется док-во от противного. Книга короткая за 2-3 дня осилишь.
Дальше попробуй доказать основную теорему о симметрических многочленах. Можешь её напрямую доказать для случая $n=2$, а затем, пользуясь результатами для этого случая, попробуй доказать для $n=3$. Сделав это, обобщи на общий случай: допустим мы доказали теорему для $n-1$, пользуясь этим докажи для $n$. Если долго не будет получаться, гугли док-во.
Дальше бери любую нормальную книгу университетского уровня по алгебре, линалу и анализу.
157 116118
>>6117
А если я такой школьник?
другой анон
158 116119
>>6118
Тебе под 30 и хочешь вкатиться в математику?
159 116120
>>6118
Ну тогда давись листками мехмата, 57 школы, ленинградскими кружками и прочим.
160 116128
>>6120

>ленинградскими кружками


>Глава 3. КОМБИНАТОРИКА-1


>Сколько разных слов в языке племени Мумбо-Юмбо?


осуждаю да ну их куда подальше негодяев
161 116132
>>6120

>листками мехмата, 57 школы, ленинградскими кружками


Особой пользы в них нет, но мозги прокачивают хорошо.
IMG2024070513392100.jpg1,9 Мб, 3120x4160
162 116133
Правильно ли я понял, что хотя в этом пункте заявлено, что мы будем искать преобразование, переводящее круг в верхнюю полуплоскость в виде функции w(z), но судя по тексту - именно z(w) является искомым преобразованием?
163 116134
>>6133
ты можешь явным образом убедиться, что куда переходит относительно заданной формулы, просто подставляя в неё явные значения (в предположении, что формула правильная)

лучше, конечно, вывести формулу самостоятельно
164 116135
Почему замкнутая кривая, образованная коническим сечением, обязана быть эллипсом? Кто-нибудь может по-быстрому написать геометрические доказвтельство?
165 116136
какие вещества принимать для диалогов с математикой?
166 116137
>>6135
потому что эта кривая задаётся квадратичным уравнением, а таких кривых не особо много, как учат в детском саду во время тихого часа аналитической геометрии

по поводу геометрической иллюстрации - представь свет фонарика, падающий на белую стену под разными углами
167 116138
>>6137

>потому что эта кривая задаётся квадратичным уравнением


Это не ответ на вопрос. Тогда почему срез конуса задается квадратичным уравнением?
Аналогию с фонариком мне тоже трудно распространить на конусы
168 116139
>>6138
потому что конус сам задаётся квадратичным уравнением

свет фонарика - это конус, световое пятно фонарика на стене - сечение этого конуса плоскостью стены
169 116140
>>6139
Спасибо большое. Затупил
170 116141
>>6136
Под веществами особо не думается, да и не ты выбираешь о чём думать, а вещество.
5a958374e1585de7719a5a211563de1e250fcc67.png3 Кб, 433x45
171 116142
Шизофреники, объясните быдлану, что означают в этой формуле n и k? И почему k=1?
172 116143
>>6142
Чето типа длины вектора от координат в $n$-мерном пространстве.
михалковакулак.jpg67 Кб, 674x710
173 116144
Предположим, что 10% населения владеют 90% совокупного богатства. Среди этих 10% распределение богатства такое же. Это можно описать гладкой функцией?
620e8b4cd0e41681397102.png23 Кб, 1300x821
174 116145
>>6143
k - это не с пикрил? Типа, k>0. И что будет с графиком при k=1?
175 116146
>>6145
Не, это вообще из другой оперы.
176 116147
>>6146
Ага, спасибо. Тогда можно ли сказать, что метрика равна квадратному корню из суммы квадратов разницы расстояний в n-мерном пространстве при количестве измерений кратном единице?
177 116148
>>6147
Только если пространство евклидово.
178 116150
>>6106

>этого дерьма уже наелся.


Скачивай задачник с прорешанными ответами и будет тебе счастье.
179 116151
>>6147

>>в n-мерном пространстве при количестве измерений кратном единице

180 116152
>>6086
посмотрел скриншот. вроде бы это то же самое решение, которое ты постил выше. оно совпадает с решением стримера, вплоть до ошибки. такое решение зачли бы в унике на бакалаврском и магистрском уровнях матспециальности
выше я высказался в общем, теперь обращу внимание на частность. ты испытываешь потребность в большем обосновании и строгости. ошибка в определении $g$ и принятие его за биекцию устранима и не нарушает идею доказательства. однако она в большей степени показательна: ты не знаешь, какие объекты можно считать существующими и какие свойства им можно приписывать
советую разобрать мое решение >>6047 и оценить его строгость. если ты посчитаешь его удовлетворительным, то это лишний аргумент в пользу моего совета читать Феферман Соломон, Числовые системы. я использовал аксиоматическую теорию множеств оттуда
у меня было решение получше, сосредоточенное на инъективности и том, что при сужении оно сохраняется, а при расширении не может возникнуть, если его не было. вроде бы это решение было попроще. я уже дважды к нему приходил, но уже не планирую к нему возвращаться
181 116153
>>6086
еще желательно использовать общие понятия, такие как сужение отображения, область значений, операции над множествами, такие как пересечение
дополнительные обозначения вводить только по необходимости
иногда лучше написать $\varphi\mid_{E_k}$ и $D(f)$, чем $\varphi \setminus \{ (k+1, \varphi(k+1)) \}$ и $E^\ast$. хотя бывает и наоборот
доказательствам от противного свойственно скрывать взаимосвязь фактов. даже если доказываешь от противного, лучше бывает расписать взаимосвязь попрозрачнее
182 116154
183 116155
>>6152

>оценить его строгость


У меня для тебя плохие новости. Чтобы ты там не напридумывал себе о "строгости" своих построений, если ты не использовал пруф-чекер, то все твои "строгие" построения - это ковыряния пальцем в жопе.
184 116156
>>6154
ну и хуйню ты высрал
185 116157
>>6156
Привет, быдло! Ну, заясни как надо.
186 116159
>>6142
Как я понимаю $p_{i}$ это $i$-ая координата точки $p$.
Всего координат $n$. Например в 3д пространстве их 3.
$n$ это число сколько координат точки у тебя есть, тебе необходимо указать что нужно брать суммы вида $p_{i}-q_{i}$ где $i$ пробегает от $1$ до $n$. Это делается значком как на твоем пике, и внизу пишется начиная с какого числа, а вверху до какого.
Вместо $i$ у тебя просто $k$ стоит.
187 116160
>>6155
все норм, не вижу плохих новостей
человек интересуется, какой смысл придать понятиям, чтобы понять, как одно следует из другого
поскольку вопрос касается содержания, а не формы, то едва ли есть польза от программы-ассистента доказательства теорем
ваше дальнейшее сравнение к теме не относится
images (1).png3 Кб, 271x186
188 116161
>>6159
Там не pi-qi, там p1-q1, причём формула с википедии и написана криво, по идее должно быть q1-p1. Потому что при таком виде формулы расстояние - это положительная разность, то бишь вычетание из большего. На схеме это правильно изображено.

>Вместо i у тебя просто k стоит.


А как это всё описать в легенде?
190 116163
>>6159
бл чувак какая i, ну ты же видишь, чел не шарит, зачем запутывать
192 116165
>>6162
>>6164

>где: i — индекс суммирования; ai — переменная, обозначающая каждый член в серии; m — нижняя граница суммирования, n — верхняя граница суммирования. Обозначение «i = m» под символом суммирования означает, что начальное (стартовое) значение индекса i эквивалентно m. Из этой записи следует, что индекс i инкрементируется на 1 в каждом члене выражения и остановится, когда i = n.[1]


О, данке шон, низкий поклон!
193 116166
>>6157

>>быдло


>>n-мерное


>> кратное единице

194 116167
>>6161
Надеюсь другой анон тебе пояснил так что ты понял.

>причём формула с википедии и написана криво, по идее должно быть q1-p1. Потому что при таком виде формулы расстояние - это положительная разность, то бишь вычетание из большего. На схеме это правильно изображено.


Ты прав, но дело в том что $(a-b)^2=(b-a)^2$ потому что $(a-b)^2=a^2-2ab+b^2$, если ты раскроешь $(b-a)^2$ получишь тоже самое, потому порядок не важен.
195 116168
>>6167
Да, спасибо, всё понятно!

>>6166
Иди нахуй, быдло.
196 116169
Как с полного нуля в математике научиться решать задачи уровня IMO? Было бы круто, если кто-нибудь составил бы подробный и проработанный список книг (желательно на русском и выпущенных не 60, а 5-20 лет назад) по алгебре, геометрии, анализу, теории чисел и комбинаторике, который было бы реально заботать за 2-3 года.
197 116170
>>6169
Это доска о математике, а не об олимпиадах.
198 116171
>>6170
Если ты или твой кумир не осилили олимпиады, то так и скажи. Реальность такова, что в какого математика не плюнь - окажется, что он выступал на олимпиадах. Например, Григорий Перельман.
199 116172
>>6171
Да-да, нужно тратить время на сборы и прорешивания идиотских задач с IMO прошлых лет, вместо изучения собственно говоря математики.

>Например, Григорий Перельман.


А Воеводский, например, нет. Ты ведь никого кроме Перельмана и не знаешь.
1) Раньше олимпиада была проще, стала сильно усложняться вконце 80-х.
2) Советские/российские математики нерелевантные примеры. Причём математики ранненго советского времени: Гельфанд, Манин, Шафаревич и тд читали книжки по анализу, теории чисел и пр, а не решали 1001 задачу на принцип дирихле, графы и прочую поебень.
Если у нас многие математики выступали на олимпиаде, то в Штатах пример обратный, большая часть не выступала и вообще довольно много тех, кто заинтересовался уже в вузе. Тот же Салливан на химика учился изначально.
3) Ты не знаешь, сколько из олимпиадников не осилило нормальную математику. Как пример тот же Савватев, любитель олимпиадных задачек, не осиливший основы АГ. Из поступивших на мат. специальность по олимпиадам в Рос. топ вузы, почти все отваливаются и укатываются в прикладники.
4) Китайских уже давно отлично выступают. Назовёшь китайских великих математиков сходу?
200 116173
>>6172
Я не понимаю, почему ты делишь математику на "олимпиадную" и "настоящую". В чём принципиальное отличие?

>Никого кроме Перельмана не знаешь


А я должен? Я обычный старшеклассник, статьи я не читаю и потому современных математиков поименно не знаю. Но я уверен, что если порыться в списке золотых призёров IMO, то больше половины там будет известными математиками.
1) Ты провёл статистические исследования или просто влепил сюда тезис с тифаретника?
2) Надо же, оказывается, в IMO нет задач по анализу и теории чисел. Ты бы хоть их компендиум почитал...
3) Савватеев не олимпиадник и не математик, он экономист, который время от времени выкладывает видео на тему вступительной и олимпиадной математики.
4) Китайцы только последние пару лет результаты показывают, до этого побеждали американцы.
Ответь на простой вопрос: что лучше — заниматься потешными кривляниями и пытаться героически изучить хотя бы программу первого курса сильнейших математических вузов, или не бежать впереди паровоза, работать олимпиадную математику и вне очереди поступить в один из этих самых вузов, получив и хороших преподавателей, и правильную среду?
201 116174
>>6173
В последнем абзаце не работать, а заботать*
202 116176
>>6173

>Я обычный старшеклассник


>2ch.hk является сайтом для лиц старше 18-ти лет. Посещая его, вы подтверждаете свое совершеннолетие.



>1) Ты провёл статистические исследования или просто влепил сюда тезис с тифаретника?


Просто смотрел задачи прошлых лет. Можешь ещё на реддит зайти в соответсвующий раздел и найти подобные вопросы и ответы.

>2) Надо же, оказывается, в IMO нет задач по анализу и теории чисел. Ты бы хоть их компендиум почитал...


А да, ты прав. Открыл вот вариант за прошлый код. Задачи на идеалы колец, многообразия, пучки.

>3) Савватеев не олимпиадник и не математик, он экономист, который время от времени выкладывает видео на тему вступительной и олимпиадной математики.


Он изначально на мехмате учился. Не потянул АГ и оформил перекат в экономисты. Что он сам не скрывает и рассказывает довольно часто.

>героически изучить хотя бы программу первого курса сильнейших математических вузов


Не нужно быть гением, чтобы освоить какую-нибудь math55, даже в одиночестве. Просто займет не 2 семетра а 2.5-3.

>поступить в один из этих самых вузов, получив и хороших преподавателей, и правильную среду?


Есть НМУ. Ну если ты школьник то дрочи олимпиады, как хочешь.
изображение.png39 Кб, 673x370
203 116177
204 116178
>>6176
Ограничение для несовершеннолетних стоит только на разделах вроде /b/, в тематике его нет.

>идеалы колец, многообразия, пучки


Какой-то набор баззвордов из оглавления учебника по global calculus. В США это graduate курс, перед ним белые люди проходят undergraduate курс по real analysis.

>несостоятельность Савватеева


Я её не отрицаю, как он вообще связан с темой разговора?

>math55


Сжатый undergraduate курс, предназначенный для математически зрелых ребят. Какой смысл скакать галопом по Европам и изучать те же многообразия и дифференциальные формы, если ты не умеешь решать нестандартные задачи по обычному calculus из тех же листков 57 школы? Чтобы умных слов нахвататься?

>НМУ


Только вот никто в одном НМУ не учится, туда ходят студенты из ВШЭ, МГУ, МФТИ и других московских вузов, которые основную информацию на лекциях получают, а в этот клоповник приходят порешать задачи и пообщаться с преподавателями.
Опять же, программа математических школ имеет большое пересечение с математикой классических олимпиад. И при подготовке к ним будет полезно тщательно разобрать книги Давидовича, Алексеева, Гашкова, Алфутова и других авторов (это всё, на минуточку, есть в рекомендациях литературы для подготовки к НМУ, можешь найти в их паблике в ВК). Мне было интересно как раз то, какие книги можно использовать для изучения теории чисел, комбинаторики, геометрии, неравенств, функциональных уравнений и прочего, потому что перечисленные выше сборники не покрывают целиком эти темы. И я спросил это здесь, потому что рассчитывал получить рекомендации, которые помогут скорректировать путь так, чтобы и на олимпиадах успешно выступить, и идейным вещам время уделить.
sage 205 116179
Помогите решить уравнение:

2^8n = n

А лучше подскажите формулы для решения, если есть такая возможность. Сам не особо математик, только начинаю вкатываться. Джва часа гуглил, ничего не нашел как решить уравнение с неизвестной в степени И неизвестной после знака равно.

С меня как обычно нихуя, но можете обоссать
206 116180
>>6167
Странно, конечно, что 22=-22, но ладно.
207 116181
>>6173

>Я не понимаю, почему ты делишь математику на "олимпиадную" и "настоящую". В чём принципиальное отличие?


тематика. этот анон опасается, что если на доске будут обсуждаться олимпиады, то ценимые им теории останутся без внимания.
>>6179
ты знаешь графики функций, задаваемых выражениями в равенстве?
208 116185
>>6178

>Ограничение для несовершеннолетних стоит только на разделах вроде /b/, в тематике его нет.


Оно на весь сайт распространяется.

>global calculus


Многообразия определяют во 2 семестре. Идеалы можно в 1.

>Я её не отрицаю, как он вообще связан с темой разговора?


Что успех на олимпиадах никак не связан со способностью изучать математику. На олимпиадах используют очень приземленные темы. Ученик спокойно может быть успешным олимпиадником на уровне всероса, но не осилить даже какие-нибудь накрытия.
На мат. направления у нас последние лет 10 поступают одни призеры всероса. Из всего потока математику осваивает несколько человек. Остальные откалываются и идут в прикладники, где вся математика за пределами линала и анализа R^n не нужна.
Гугли, например, письмо студентов ВШЭ. Большинство было против изучать собственно говоря математику, а хотело кушать более приземленные вещи, благодаря которым можно потом получать кучу денег. Зачем только они пошли на "математику" не ясно. Именно математику хотело изучать меньшая часть студней, написавших письмо.

>если ты не умеешь решать нестандартные задачи по обычному calculus из тех же листков 57 школы?


Почему-то к калькулюсу у людей особое отношение. Давай подставив вместо него например теорию групп. Зачем мне прорешивать нестандартные задачи, аля "найти все группы порядка 16"? Это только займёт моё время, которое лучше потратить на что-то содержательное. В анализе заковыристые задачи можно составлять бесконечно, заворачивая их в трюки, и их решит только тот кто эти трюки знает. Но это бессодержательно, эти трюки тебе дают ничего в изучении последующих дисциплин.

>для изучения теории чисел, комбинаторики, геометрии, неравенств, функциональных уравнений и прочего


Я тебя наверное расстрою, но все эти темы тебе больше никогда не понадобятся. Особенно геометрия и особенно функциональные уравнения, которые и существуют лишь на олимпиадах. Теория чисел тебе нужна в объеме основной теоремы арифметики, комбинаторика в объеме уметь расставить коэффициенты в биноме Ньютона.

Короче делай что хочешь, мне похуй, переубеждать тебя у меня цели не стоит. Я высказал своё мнение.
208 116185
>>6178

>Ограничение для несовершеннолетних стоит только на разделах вроде /b/, в тематике его нет.


Оно на весь сайт распространяется.

>global calculus


Многообразия определяют во 2 семестре. Идеалы можно в 1.

>Я её не отрицаю, как он вообще связан с темой разговора?


Что успех на олимпиадах никак не связан со способностью изучать математику. На олимпиадах используют очень приземленные темы. Ученик спокойно может быть успешным олимпиадником на уровне всероса, но не осилить даже какие-нибудь накрытия.
На мат. направления у нас последние лет 10 поступают одни призеры всероса. Из всего потока математику осваивает несколько человек. Остальные откалываются и идут в прикладники, где вся математика за пределами линала и анализа R^n не нужна.
Гугли, например, письмо студентов ВШЭ. Большинство было против изучать собственно говоря математику, а хотело кушать более приземленные вещи, благодаря которым можно потом получать кучу денег. Зачем только они пошли на "математику" не ясно. Именно математику хотело изучать меньшая часть студней, написавших письмо.

>если ты не умеешь решать нестандартные задачи по обычному calculus из тех же листков 57 школы?


Почему-то к калькулюсу у людей особое отношение. Давай подставив вместо него например теорию групп. Зачем мне прорешивать нестандартные задачи, аля "найти все группы порядка 16"? Это только займёт моё время, которое лучше потратить на что-то содержательное. В анализе заковыристые задачи можно составлять бесконечно, заворачивая их в трюки, и их решит только тот кто эти трюки знает. Но это бессодержательно, эти трюки тебе дают ничего в изучении последующих дисциплин.

>для изучения теории чисел, комбинаторики, геометрии, неравенств, функциональных уравнений и прочего


Я тебя наверное расстрою, но все эти темы тебе больше никогда не понадобятся. Особенно геометрия и особенно функциональные уравнения, которые и существуют лишь на олимпиадах. Теория чисел тебе нужна в объеме основной теоремы арифметики, комбинаторика в объеме уметь расставить коэффициенты в биноме Ньютона.

Короче делай что хочешь, мне похуй, переубеждать тебя у меня цели не стоит. Я высказал своё мнение.
209 116186
>>6178
У тебя какой-то олимпиадный маниакал походу. Олимпиада - это спорт. Олимпиадное движение в математике построено точно так, как и олимпиадное движение в спорте - то есть как система фильтров. На вход воронки подается миллионная подростковая биомасса, чтобы на выходе получился один чемпион мира, девятьсот тысяч неудачников и сто тысяч калек. Как профессиональный спорт не делает человека здоровее, так и олимпиады не делают людей умнее - они просто прогоняют биоматериал по слоям из узкоспециализированных обучающих и тестовых выборок и отсеивают индивидов с высоким айку, нанося всем не прошедшим отбор глубокую психологическую травму.

Как и в спорте, без финансирования, тренера и команды ты какашка. Поэтому в нынешних олимпиадах побеждают не столько самые умные, сколько жители мегаполисов. Так что самый лучший совет начинающему олимпиаднику - перезачать себя заново и родиться москвичом. Только вот при чем тут математика?

Олимпиады начинались в качестве приятных бонусов и оргмероприятий третьестепенной важности, посвященных популяризации математики. Школьники просто сидели и изучали нормальную математику, а на сдачу бегали на олимпиадные баттлы. Но совок раздул значимость этой периферийной хуйни до галактических масштабов. А введение БВИ по результатам олимпиадных достижений превратило эту хуйню в добровольно-принудительную повинность.

И опять же, если ты обычный нищий рассеянский холоп, которому хоть тушкой хоть чучелом нужно заскочить в ВШЭ на бюджет - то при чем тут, блядь, математика? Математики изучают математику и параллельно решают задачи, имеющие непосредственное отношение к изучаемому предмету. А вот батареи синтетических тестов, жирные рукшины-педофилы, реноме краснопузой державы и золотые собачьи медали - это где-то совсем в другой стороне.
210 116187
>>6185
>>6186
Вербитофаги, как всегда — пришли, пёрнули в лужу и ушли.
211 116190
>>6187

>Вербитофаги


кто это?
212 116193
>>6190
Такие вот забавные ребята, нитакиекакфсе, которые наслушались этого фрика и побежали везде разносить его мысли.
213 116195
>>6193
Этот фрик сейчас с тобой в одной комнате? Расскажи за какие места он тебя трогал.
изображение.png19 Кб, 806x229
214 116196
>>6187
>>6193
На /r/math/ чатсо задают вопросы по поводу олимпиад, и ответы там не в пользу них. Это что, тоже фанаты тифарета, сидят и через переводчик его шизу о политике читают?
Например
https://www.reddit.com/r/math/comments/vznzlm/whats_your_opinion_on_mathematics_olympiads/
https://math.stackexchange.com/questions/1883563/is-being-good-at-mathematic-contests-necessary-to-pursue-a-career-in-mathematics
215 116197
>>6196
да
216 116198
>>6196
Во 2 ссылке кстати и мнение Терри Тао, золотого медалиста IMO
https://terrytao.wordpress.com/career-advice/advice-on-mathematics-competitions/

>But mathematical competitions are very different activities from mathematical learning or mathematical research; don’t expect the problems you get in, say, graduate study, to have the same cut-and-dried, neat flavour that an Olympiad problem does

217 116199
>>6196

>шахматы бесполезны


Это многое говорит об интеллектуальных возможностях этого человека.
>>6198
Это противоречит тому, что решение олимпиадных задач полезно? Нестандартные задачи учат творческому и креативному мышлению. Или ты из тех сектантов, кто считает, что олимпиады можно покорить, если заучить набор неких трюков?
изображение.png139 Кб, 608x866
218 116201
>>6199
А нормальная математика мыслить креативно не учит?

>Или ты из тех сектантов, кто считает, что олимпиады можно покорить, если заучить набор неких трюков?


Так ты же сам полный 0, о чём ты писал в первом посте. Схуяли ты такие суждения делаешь? Ну предположим ты другой анон.
Конечно просто заучив трюки ты не решишь задачи. Но если ты эти отрюки отработаешь на кучу других задач, то решишь многие в бою. Потому что все олимпиадные задачи и строятся таким образом, задачу составляют заранее зная трюк, которым необходимо её решить. Более сложные задачи миксуют трюки. И набор трюков заранее известен.
Если ты откроешь любую книжку по олимпиадной математике для вкатунов, то её главы и есть названия этих трюков. Например пик - "The art of problem solving". Зная эти трюки попав на олимпиаду у тебя уже дикое примущество над тем, кто может, более склонен к математике, чем ты, но эти трюки он не знает.
И, как следствие, олимпиады даже не развивают, а ухудшают творческое мышление, потому что во всех задачах ты начинаешь искать эти слабые места, тыкнув в которые задача развалится. Но реальные задачи решаются совсем иначе. О чём например и цитата Тао выше. Кто-то от этой привычке может избавиться и нормально развиваться дальше, а кому-то она ломает мозг на всю оставшуюся жизнь. Потому лучше не рисковать и держаться от олимпиад подальше.
219 116203
>>6196

>фанаты тифарета


А их что много?
image.png31 Кб, 1116x112
220 116204
Мужики, проверьте доказательство пж.

Пусть у $ E $ нет предельных точек в $ \mathbb{R}$, тогда для любой точки $ x \in E$ найдётся такая проколотая окрестность $ U'(x) $ такая, что $ U'(x) \cap E = \varnothing $. Ясно, что такие окрестности можно сделать попарно непересекающимися для любой точки из $E$. Тогда можно рассмотреть взаимнооднозначное соответсвие между точками множества $E$ и такими проколотыми окрестностями точек из $E$. Но любое множество непересекающихся интервалов на прямой является не более чем счётным множеством. Противоречие с тем, что $ E$ континуально.

Верно ли решение?
221 116205
>>6204

>Ясно, что такие окрестности можно сделать попарно непересекающимися для любой точки из E


следует подчеркнуть, что речь идёт об $\mathbb R$ потому что в общем случае это неверно

>Но любое множество непересекающихся интервалов на прямой следует пояснить, что твои окрестности сами представляют собой объединения непересекающихся интервалов, тем самым аргумент к можно применять

222 116206
>>6205
согласен
223 116208
>>29047 (OP)
Комбинаторика, помогите понять где ошибка в рассуждениях.
Задача:
10 студентов, среди которых С. Федин и А. Шилов, случайным образом занимают очередь в библиотеку. Сколько имеется вариантов расстановки студентов, когда между Фединым и Шиловым окажутся 6 студентов?
Ответ из книги:
В ответе 328!, как я понимаю, логика такая: Шилов и Федин отстают друг от друга на 7 позиций (если Федин на i-ой позиции, то Шилов на i-7 или i+7). Допустим, Шилов первый, таких вариантов может быть 3 - (1, 8), (2, 9), (3, 10) (3 в ответе). Также, Шилов и Федин могут поменяться местами (2 в ответе). И 8! это перестановка для всех остальных студентов.
Моё решение:
Объединяем Федин + 6 студентов + Шилов в один "блок"/"студента". Получается 3 студента (2 чела + этот блок), можем переставить их 3! способами. В блоке мы можем переставить людей между Шиловым и Фединым 6!. +переставить самих Федина и Шилова. Получается 3!26!. Где ошибка?
224 116209
>>6199
Ты отбитый же наглухо. Креативному мышлению учит длительная (от трех месяцев и более) автономная исследовательская работа, в ходе которой человек самостоятельно парсит теоретическую бигдату, формулирует проблематику и ставит себе задачи - потому что мышление, блядь, это по определению марафон. Ровно так же как олимпиады это по определению спринт - когда тебе за фиксированное количество времени нужно забрутфорсить энигму с помощью автокомплита в виде пары сотен задроченных эвристик и двух-трех спонтанных инсайтных скачков.

Внутри отечественной кружково-олимпиадно-матшкольной культурки, за редчайшими исключениями, никакая математика и не плавала - там сплошняком идет применение эвристик к потешным конструкциям низкой размерности внутри т. н. "сюжетов". Уровень олимпиад отличается количеством требуемых для их прохождения эвристик. Именно для этого командам нужен тренер, который проанализировал архивы задач за прошлые годы, примерно догадывается, что там будет в следующем году - и исходя из этого подбирает пул эвристик, более-менее равномерно покрывающих весь спектр гипотетически возможных конструкций. И именно поэтому в олимпиадах чаще всего побеждают посредственные москвичи, а не умные провинциалы - потому что у первых есть доступ к 200 авторским трюкам, а вторые ограничены трюковой базой, почерпнутой из классических книжек.

Ну да, можно теребить алфутову и полагать, что занимаешься математикой. Можно играть в куклы и думать, что завел семью. Можно возить машинку на веревочке и считать себя ниибаца водителем. Можно круглосуточно играть в шахматы и считать, что возможен трансфер полученных навыков из шахмат в домены других игр (нет). Еще можно путать корреляцию с каузацией и считать победы в олимпиадах причиной математических достижений Перельмана - хотя на самом деле все его победы и достижения обусловлены одним и тем же фактором: аномально высоким уровнем интеллекта.
225 116212
>>6201
>>6209
приведите, пожалуйста, более или менее обширный пример того, как решение задач основано на трюках
пучть будет перечень трюков и задания 4-5 олимпиад, и чтобы с помощью трюков можно было набрать значительное число баллов
226 116214
Зря с зашёл в этот клоповник, в общем. Поищу лучше сам, всяко лучше, чем ждать ответа от местной "интеллектуальной элиты".
227 116215
>>6208
Ты не учел, что 6 студентов, которых ты объединяешь в блок с Силовым и Фединым, можно выбрать многими способами, а именно биномиальный коэффициент С(2, 8). Домножь свой ответ на него, и получишь совпадение с книжным ответом.
228 116217
>>6215
Точняк, спасибо.
229 116226
>>6185

>Гугли, например, письмо студентов ВШЭ. Большинство было против изучать собственно говоря математику, а хотело кушать более приземленные вещи, благодаря которым можно потом получать кучу денег. Зачем только они пошли на "математику" не ясно. Именно математику хотело изучать меньшая часть студней, написавших письмо.



https://vk.com/doc137862366_374777111?hash=LaqCbxlMERjb6gpnxUXuiYlHu4fB8eZ8NL38HCWvyYH&dl=3MAgEK7xh8Tx1mLswvb1Vmai9S0BIp3QePgycB6jcGX

???
230 116227
>>6226
Что ты хотел узнать? Оно ли это? Да. Есть ещё несколько видео c очного обсуждения, где зачем-то позвали Делиня, не владеющего русским, который сидел и 1.5ч нихуя не понимал.
https://www.youtube.com/watch?v=b0WTyetuV6Y
https://www.youtube.com/watch?v=a7ZfClXj6-k
Ну и по письму можно оценить, насколько же ахуенно преподают в "топ" местах в России. От этого письма ничего не поменялось в итоге.
231 116228
>>6227

>где зачем-то позвали Делиня


и вдобавок анон когда-то даже сочинил пасту, не могу не вспомнить

>Миша, привет. Пишет тебе Пьер Делинь из Института перпекстивных исследований в Принстоне. Я много времени не займу. Просто хочу сказать, что ты ебаный мудак, кусающий руку, которая тебе кормит. Ты говно, понял?


Никому не нужна твоя "мудацкая первая культура". Это все пустое. Студентов ВШЭ должны учат дифференциальным уравнениям и механики. Недавно меня приглашали на заседание по реформированию программы ВШЭ, и там все говорили правильно. Диффуры и механика - нужны. Это науки будущего. Подошел к концу век пучков и когомологий.

>Ты, говно такое, промыл мозги юным и впечатлительным студентам ВШЭ, и теперь они верят, что "диффуры - говно, пучки - хорошо". Нет, Миша. Говорю тебе, как человек, всю жить занимавшийся алгебраической геометрией. Моим наставником был Александр Гротендик. Все, лавка закрывается, Миша. Гротендик, я, ты, Серр, Бурабки - это прошлый век. Руководство ВШЭ это понимает и учит студентов наукам будущего - дифференциальным уравнениям, механике, теории вероятностей.


>Люди понимают, что абстрактная математика не нужна налогоплательщикам, человечеству нужна та математика, которая произведет революцию в прикладных дисциплинах. Нужна математика, которая реформирует мировую финансовую систему, чтобы больше не было нуждающихся, нужна математика, которая даст нам вылететь за пределы галактики, нужна математика, которая поможет нам контролировать погоду.


>Твои птенцы тут смеялись, что я, "старый хрыч", всё заседание сидел, "как дурак", и ничего не понимал, просто кивал. А я понимал. Я понимал, что потратил жизнь на никому не нужную абстракщину. И понимал, что не могу допустить, чтобы студенты ВШЭ пошли по моим стопам. Я абсолютно убежден, что ученые умы "Вышки" правы. Что нужны прикладные физические и аналитические науки. Это - будущее математики. Это - будущее человечеста. Это - будущее ВШЭ.



>Пошел нахуй, дрочила чертов.



>С уважением,



>Пьер. Институт перспективных исследований, Принстон, Нью-Джерси.

231 116228
>>6227

>где зачем-то позвали Делиня


и вдобавок анон когда-то даже сочинил пасту, не могу не вспомнить

>Миша, привет. Пишет тебе Пьер Делинь из Института перпекстивных исследований в Принстоне. Я много времени не займу. Просто хочу сказать, что ты ебаный мудак, кусающий руку, которая тебе кормит. Ты говно, понял?


Никому не нужна твоя "мудацкая первая культура". Это все пустое. Студентов ВШЭ должны учат дифференциальным уравнениям и механики. Недавно меня приглашали на заседание по реформированию программы ВШЭ, и там все говорили правильно. Диффуры и механика - нужны. Это науки будущего. Подошел к концу век пучков и когомологий.

>Ты, говно такое, промыл мозги юным и впечатлительным студентам ВШЭ, и теперь они верят, что "диффуры - говно, пучки - хорошо". Нет, Миша. Говорю тебе, как человек, всю жить занимавшийся алгебраической геометрией. Моим наставником был Александр Гротендик. Все, лавка закрывается, Миша. Гротендик, я, ты, Серр, Бурабки - это прошлый век. Руководство ВШЭ это понимает и учит студентов наукам будущего - дифференциальным уравнениям, механике, теории вероятностей.


>Люди понимают, что абстрактная математика не нужна налогоплательщикам, человечеству нужна та математика, которая произведет революцию в прикладных дисциплинах. Нужна математика, которая реформирует мировую финансовую систему, чтобы больше не было нуждающихся, нужна математика, которая даст нам вылететь за пределы галактики, нужна математика, которая поможет нам контролировать погоду.


>Твои птенцы тут смеялись, что я, "старый хрыч", всё заседание сидел, "как дурак", и ничего не понимал, просто кивал. А я понимал. Я понимал, что потратил жизнь на никому не нужную абстракщину. И понимал, что не могу допустить, чтобы студенты ВШЭ пошли по моим стопам. Я абсолютно убежден, что ученые умы "Вышки" правы. Что нужны прикладные физические и аналитические науки. Это - будущее математики. Это - будущее человечеста. Это - будущее ВШЭ.



>Пошел нахуй, дрочила чертов.



>С уважением,



>Пьер. Институт перспективных исследований, Принстон, Нью-Джерси.

232 116229
>>6228
Обпучкался с пасты.
233 116230
>>6227
Так там же тогда написано совершенно противоположное тому, что написано в этом >>6185 посте.
234 116231
>>6230
Я просто криво написал. Но читай концовку

>Именно математику хотело изучать меньшая часть студней, написавших письмо.


Под >большинством я имел ввиду всех других, письмо не подписавших и котоые нежелали менять программу, потому что "не все хотят быть теоритками пучков", фраза откуда-то из двух видео что я предоставил.
235 116232
>>6231
Ладно.
Но с тех пор по рассказам и по тем конспектам, что я вижу, там всё же многое изменилось.
236 116233
>>6228
А у пучков есть применения в прикладной математике?
237 116234
>>6214
Правильный выбор. Тут петухи одни, кукарекающие на разные лады у параши.
238 116236
>>6233
должны быть - наверняка есть задачи, которые сводятся к анализу на комплексных многообразиях, а там уже и пучки, и всё, что душа пожелает

я за дискуссией выше не следил
239 116237
>>6234

>Тут петухи одни, кукарекающие на разные лады у параши.


А ты самокритичен.
21341.png123 Кб, 469x584
240 116239
Может кто-нибудь объяснить каким образом и откуда получилось последнее неравенство? Спрашиваю чисто из любопытства, потому на работе от нечего делать. Левая часть - понятно квадрат суммы и h выносим за скобки.
Это У. Рудин 10 стр.
241 116240
>>6239
в последней строчке два неравенства
из них первое - в силу того, что $h < 1$, второе - что $h$ меньше указанной дроби в формуле выше справа
242 116243
>>6239
Рудина тяжело читать, как пример здесь он берет числа снихуя и просто подставляет. Ну ахуеть, а взялись они откуда? И так везде. Я равенства эти доказывал так
Допустим $p$ крайнее число такое, что $p^2<2$. Проверим нет ли других, добавив очень маленькое $0<h<1$
$(p+h)^2=p^2+2ph+h^2 < 2$
$2ph+h^2 = h(2p+h) < 2 - p^2$
$h < \frac{2-p^2}{2p+h}$
получаем странное неравенство. Чем больше знаменатель справа, тем меньше дробь в целом, и нам нужно выбрать такое число, что меньше этой дроби. Тк $h<1$, то максимальный знаменатель при $h=1$
$h < \frac{2-p^2}{2p+1}$
Проверим подставив такое $h$
$(p+h)^2=p^2+2ph+h^2=p^2+h(2p+h) < p^2+\frac{2-p^2}{2p+1}(2p+h)$
здесь можно попасть в ловушку, начав приводить к общему знаменателю, поэтому схитрим обратив внимание, что
$\frac{2-p^2}{2p+1}(2p+h)=(2-p^2)\frac{2p+h}{2p+1}$
ясно что $2p+h<2p+1$, это следует из того что $h<1$, а из этого следует что эта дробь меньше $1$, мы меньшее делим на большее. Значит и всё произведение меньше чем $2-p^2$ Тогда
$(2-p^2)\frac{2p+h}{2p+1}$<(2-p^2)1=2-p^2$ возвращаемся к основному неравенству
$p^2+\frac{2-p^2}{2p+1}(2p+h)<p^2+2-p^2=2$

Но можно как Рудин сразу заменить $2p+h$ на $2p+1$ и просто сократить дробь.

Рудин слишком сухой. Я перекатился на Зорича. Потом можешь Шварцем шлифануть, хотя не обязательно.
243 116245
Прочитал "обыкновенные дифуры" Арнольда, абсолютно охуенная книжка. Ищу нечто похожее про уравнения в частных производных, может аноны знают что. Можно на англ.
244 116250
>>6245

>Прочитал "обыкновенные дифуры" Арнольда, абсолютно охуенная книжка.


Чем же она так хороша? Что узнал из нее нового?

>в частных производных


У Арнольда и про них есть.
245 116282
>>6250
Так рекомендации будут или нет?

"Есть" это один пример, две сноски, и три задачки. Считай что и нет ничего. Про тот же лапласиан можно тома целые писать. Напомни нам, сколько в этой книжке про лапласиан? Одна задача. Что не удивительно, ведь книжка про ОДУ.

Хороша тем, что даёт интуитивный взгляд для математиков. Ну мне во всяком случае зашла лучше, чем стандартные книжки с хуитой на механическое решение. Поэтому и ищу нечто похожее для уравнений в частных производных. Традиционные рекомендации (Evans) уже прочтены.
246 116283
>>6282
Ты наркоман нахуй?
http://www.vixri.ru/d2/Arnold V.I. _Differ. Uravnenija v Chastnyx Proizvodnyx.pdf
На мои вопросы будешь отвечать?
247 116285
>>6282

>Традиционные рекомендации (Evans) уже прочтены.


Читай Хермандера, чего мелочиться
248 116289
>>6283
Я про его ОДУ книжку шиз, таблетки ешь раз буйный такой. Я даже не знал, что у него есть про частные, замечательно, спасибо, буду изучать.

>>6285
Нужно с разных сторон читать для настоящего понимая, ящитаю
249 116290
>>6289
Чет ты больно дерзкий для такого безмозглого чмошника. Тебе так не кажется? Но подкину тебе одну идейку, мне не жалко. Арнольд то за жизнь не одну книжку высрал. Так что глянь его библиографию. Хуй знает что там тебе понравилось в его книге (ты так и не раскрыл эту тему, ожидаемо), но может и в других его трудах чего приглянется.
250 116292
>>6289
для настоящего понимания надо работать в этой области, а не просто читать (хотя если ты поймёшь Хермандера, можешь уже считать себя крутым экспертом - это не всякому под силу)

на самом деле, если ты и в самом деле прочёл всю книгу Эванса, то твой вопрос удивляет: пора бы уже знать, какие у тебя цели и что именно надо читать, чтобы к ним двигаться; если же ты осилил её только развлечения ради и хочешь продолжать, то следующий логичный шаг (на мой взгляд) это эллиптические операторы на многообразиях и атья-зингер. Дерзай
251 116295
>>6290
хуя у тебя тряска

>>6289
эванс тот еще кал кстати
252 116296
>>6295
Чмоха, тебя похоже самого потрясывает вот тебе и мерещится всякое.
253 116305
>>6185

> поступают одни призеры всероса


> Остальные откалываются и идут в прикладники


Это не потому что они не могут её осилить, а как раз потому что они в среднем умнее: сразу просекают какая гойская тема (для понятно какого большинства студентов) эта профессиональная математика, да и академическая карьера в целом, и идут заниматься чем-то полезным для себя
254 116306
>>6201

>реальные задачи решаются совсем иначе


Трюки эти на самом деле в большой своей части придумываются не из воздуха, а и берутся из "настоящей математики". На твоём же скрине, например, указаны симметрии и инварианты в качестве "тактик". Искать симметрии и инварианты это основа математики
255 116309
>>6306
Странно только, почему составители задачников и тренеры IMO команд не какие-то великие математики, а хуй пойми кто, и почему те кто реально ею занимаются говорят, что олимпиады и собственно говоря математика, очень сильно отличаются.

>Искать симметрии и инварианты это основа математики


В этом и проблема олимпиад, что она мозг перестравает так, что он пытается найти известный сюжет. Но реальные задачи не так решаются.
256 116310
Аноны, в FAQ очень много книг, но интуиция подсказывает мне, что они пиздецки углубляются в математические дебри. Что можете посоветовать более поверхностного для вчерашних школьников-сегодняшних студентов нематематических специальностей?

Короче говоря, база матана и лин. алгебры, чтобы ориентироваться и ПОНИМАТЬ
257 116311
>>6310

> но интуиция подсказывает мне


возьми и начни читать
не покатит - возьми другое
по базовым дисциплинам учебников сотни, и найти среди них что-нибудь, отвечающее твоим вкусам, всегда возможно если ты не петух-неосилятор: его удовлетворить не может ничто в принципе; но это не потому, что все учебники плохие
258 116313
>>6310
По матану Зельдович, но лично для меня он тяжело написан, читал Шварцбурда Анализ для ПТУ. Можно Фихтенгольца, но меня пугало колличество страниц в школе.
Линал уже сложнее, хотя предмет проще пареной репы. Самые хорошие книжки по линалу это Linear algebra via exterior product, Булдырев-Павлов и Глазман-Любич. Но я не знаю как они пойдут у вкатуна, я учился по 2 и 3. Можешь что-то дефолтное типа Гельфанда или Кострикина взять. Можешь Акслера наверное. Можешь Дьедонне попробовать.
259 116315
>>6181

>ты знаешь графики функций, задаваемых выражениями в равенстве?



Не совсем понял. Предлагаешь превратить выражение в функцию и нарисовать график?
По сути вопроса - не очень и да изначальный вариант предполагает графики (данное уравнение - это сокращение), как я понял я ошибся малясь с ними, но все равно не понимаю как это должно помочь глобально с поиском n неизвестной я ужасный нуб
cube.png82 Кб, 1170x2048
260 116316
Сап, дневной.

Помогай. Мне задали задачку на паре по комбинаторике.

В условии написано, что надо узнать, цитирую:

Представьте себе многомерный куб (гиперкуб) с N измерениями (рис. 1). Соединим все вершины этого куба линиями, чтобы получить полный граф. Теперь давайте раскрасим каждую из этих линий (рёбер графа) в один из двух цветов: красный или синий.

Вопрос: при каком наименьшем значении N (количества измерений гиперкуба) мы гарантированно получим нарисованную одним цветом группу из четырех вершин, лежащих в одной плоскости? (как на рис. 2)
261 116317
>>6316
Пробовал преобразование Фурье применить?
262 116318
>>6315
я имею в виду не столько сами графики, сколько возрастание и скорость возрастания функций. просто графики наглядно это отображают.
в левой части экспоненциальная функция, а в правой - линейная. они обе возрастают, но экспоненциальная ускоряет рост, а линейная растет равномерно. значит, пересечься графики могут не более двух раз.
точки пересечения можно подобрать со сколь угодно высокой точностью. например, методом вилки.
в данном случае $f(x) = 2^{8 \cdot x}$ всегда положительна, поэтому на $(- \infty, 0]$, где $f(x) = x$ не выше 0, пересечений не будет. в 0 тоже экспоненциальная больше линейной, и уже производная экспоненциальной выше. так что пересечений не будет и дальше.
263 116319
>>6309
Так и решаются, технически, как они по-твоему в принципе могут решаться, с помощью магии и божественного прозрения?
264 116320
>>6311
Ладно. По первым главам уже, по идее, можно будет составить своё мнение. Всё-таки что-то я ещё помню

>>6313
Спасибо, сохраню. У Зельдовича, вроде, интересная книга
265 116321
>>6319

>как они по-твоему в принципе могут решаться, с помощью магии и божественного прозрения?


С помощью преобразования Фурье.
266 116322
>>6243
Немного понял, спасибо за объяснение
267 116323
>>6316
"Догадайся", что наименьшее такое N равно 5. Докажи, что в полном 5-кубе обязательно найдётся монохромный квадрат. Выбери вершину, обозначим ее v_0. При этой вершине 31 рёбер, из них по крайней мере 16 одного цвета, без ограничения общности, пусть будет красного. Обозначим вершины, соединённые этими красными ребрами с v_0, как v_1,...,v_16. Возьми v_1, рассмотри 15 ребер из v_1 в v_2,...,v_16. Сколько из них должны быть одного цвета? Покажи, что в меньших измерениях не найдётся. Закрась ребра так, чтобы не было ни одного монохромного квадрата.
268 116324
>>6319
Ну, раз уж этого достаточно, то вперед гипотезу Римана доказывать.
269 116325
>>6322
Я еблан. На самом деле можно проще, сейчас присмотрелся.
Начинаем так же. Допустим $p$ крайняя точка, прибавим к нему очень маленькое чиселко $0<h<1$
$(p+h)^2=p^2+2ph+h^2=p^2+h(2p+h)$
Не трудно заметить, что $2p+h < 2p+1$, тогда
$p^2+h(2p+h)<p^2+h(2p+1)$
у нас есть свобода в выборе $h$, потому мы можем избавиться от этой скобки, взяв дробь со знаменателем $2p+1$
$p^2+\frac{h}{2p+1}(2p+1)=p^2+h$
остается выбрать такое $h$, что $p^2+h<2$, и тут уже и школьник справится
$h<2-p^2$
270 116327

>every net has some subnet that is an ultranet, but no nontrivial ultranets have ever been constructed explicitly


лол
лмао
271 116328
>>6327
Впервые аксиому выбора видишь?
272 116331
>>29047 (OP)
няшки, какие ресурсы используете для упрощения и объяснения выражений?
Пробовал
https://www.symbolab.com/solver/simplify-calculator
но он платный, еще другие то же платные.
Памагите111
273 116333
Очень хочется после просмотра научпоп-контента попробовать изучить геометрию многообразий. В интернете пишут, что перед этим нужно сначала изучить математический анализ и линейную алгебру. Тут есть два пути. Первый путь — "Математический анализ I/II" В.Зорича + "Курс алгебры" Э.Винберга, второй путь —"Analysis I/II/III" H.Amann и J.Escher. Мне больше нравится второй вариант, потому что больше контента, но он на английском, который я вообще не знаю, к тому же, как мне кажется, порог входа там выше, а я из математики знаю только квадратные уравнения, даже теорему Пифагора самостоятельно не докажу. Так как же мне всё-таки поступить?
274 116334
>>6333
нравится второй вариант - следуй по второму варианту

"больше контента" - в случае мат. анализа это аргумент скорее против, чем за: какой там может быть контент?
для гладких многообразий от анализа по-настоящему нужна только теорема о неявной функции
местами ещё полезны: формула Тейлора, интегрирование, теорема о единственности решения задачи Коши для ОДУ первого порядка.
в принципе можно считать, что указанные темы как раз исчерпывают весь мат. анализ

английский надо изучать, если хочешь заниматься математикой, поэтому читать на английском лучше, чем на русском

И самое главное - зачем тебе эти многообразия нужны? если ради только развлечения (а ты знаешь теорему только теорему Пифагора), лучше на них не смотреть
275 116335
>>6334
У русских учебников побольше комьюнити, мне ведь будут нужны решения задач, поэтому и спрашиваю. А геометрия многообразий мне видится наукой, которая позволяет формализовать сложные геометрические объекты для упрощённой работы с ними. Я думаю, что это используется в физике, которая была интересной для меня до того, как совковый препод привил мне ненависть к этому предмету. Меня вообще мало привлекают абстрактные вещи вроде алгебры, которую тут очень уважают, судя по всему. А вот то, что можно представить и нарисовать, мне доставляет.
276 116336
>>6335
многообразия используются в физике
алгебра используется в математике для исследования самых разных геометрических вопросов, поэтому она нужна, хотя для гладких многообразий в значительно в меньшей степени; линейная алгебра нужна везде в полном объёме (хотя она сама очень легко представляется в геометрической интерпретации, потому упрекать её в излишней абстрактности не приходится)
277 116337
>>6335
Читай учебники Арнольда по механике.
278 116338
>>6335
Ну и по поводу алгебры, учебник Винберга немного с уклоном в физику, насколько это вообще возможно для учебника по алгебре на таком уровне. См. еще книжку Артина про геометрическую алгебру.
279 116339
>>6335
Я вкатился из-за любви к алгебре, но потом её разлюбил. Современная алгебра берет своё начало с конца классической алгебры с решениями уравнений. Вот начало интересное. А дальше идёт ебанина, котору навыдумывали для решения ВТФ и так и не ришили. А вот когда эту ебалу стали прикладывать к геометрии, получилось снова интересно. Вообще науки которые сами в себе это всегда унылое зрелище. Какие-то основы можно подучить, но в дебри лезть уже нужно аутизмом обладать. Интересные вещи они всегда на стыке.
280 116340
>>6333
сорян если не в тему но зач многообразия
можно изучать конструктивные объекты, натуральные числа, вообще какие-то культурно ценные и жизненные понятия
281 116341
>>6337
Слишком сложно. Я скорее всего начну с Зорича, потому что у него всё с полного нуля.
282 116342
>>6331
Вольфрам же. Но чтобы step by step решение увидеть нужно платить.
283 116343
>>29047 (OP)
Сап, матач. Есть вопросик.
Имеется массив наблюдений. Нужно проанализировать временной ряд и спрогнозировать методом Фурье возможные будущие значения наблюдаемой величины. Почитал в интернете, вроде алгоритм понятный, но как именно применить его на практике - загадка. Мб посоветуете литературы какой на этот счёт, или, если не сложно, объясните...
284 116344
>>6243
>>6325
>>6322
У нас есть два неравенства сверху, их напрямую применили снизу. Всё.
285 116345
>>6342
Есть такая вещь под названием пиратство.
286 116346
>>6333
Что такое геометрия многообразий конкретно? Теория гладких многообразия? Теория алгебраических многообразий? Дифференциальная геометрия? Геометрическая топология?
287 116347
>>6338

>учебник Винберга немного с уклоном в физику


???
288 116348
>>6339

>А дальше идёт ебанина, котору навыдумывали для решения ВТФ и так и не ришили.


Так решили же.
Вообще, современный алгем (а точнее арифгем), несмотря на всю кажущуюся абстрактность, всё ещё про уравняшки и естественным образом развился из преодоления недостатков подходов прошлого. Это https://arxiv.org/abs/math/0508174 один из моих любимых примеров того, что вся машинерия современной алгебры необходима для исследования одного конкретного уравнения.
289 116349
>>6345

>>Вольфрам же. Но чтобы step by step решение увидеть нужно платить.


>Есть такая вещь под названием пиратство.


предлагаешь заплатить шиллингами и остаться анонимным?
290 116350
>>6346
Теория гладких многообразий. Зорича для неё хватит?
291 116351
>>6349
Предлагаю не платить и пользоваться step by step бесплатно.
292 116352
>>6347
Есть немного про группы в физике, линал излагается с ориентацией на нужды физиков (имхо), есть глава про группы Ли.
293 116353
>>6345

>пиратство


А Вольфрам разве есть в офлайн версии?
294 116354
>>6310
по линейке мне книга linear algebra done wrong зашла.
295 116355
>>6350
Все классические курсы матана плюс-минус похожи. Весь Зорич тебе, конечно же, не нужен. Тебе из всего курса нужен анализ на R (от пределов до дифференцирования и интегрирования, многочлен Тейлора; ряды для гладких многообразий не нужны вообще, например) и анализ на R^n (дифференцирование, интегрирование, теорема о неявной/обратной функции она самая важная), там тебе немного топологии метрической дадут ещё. Тебе, в принципе, об этом написали уже. Это примерно первые 2 семестра, хотя интегрирования уже в третьем обычно.
Курс алгебры это оверкилл, весь Винберг на 2-3 семестра на математическом факультете рассчитан, тебе же нужны лишь некоторые элементы линейной алгебры. Но книжка дюже хорошая для поднятия математической культуры (хотя в основном алгебраической). Для твоих целей можно до тензорной алгебры включая почитать, скипнув проективные пространства и, возможно, алгебру многочленов (но я бы почитал, если торопиться некуда, глава хорошая). Про группы лучше хоть что-то да знать, поэтому лучше не пропускать главу по ним. Про тензорное и внешнее произведение тоже. Впрочем, обычно во вступительных книжках по многообразиям их пытаются по-быстрому с нуля объяснить (но чаще всего в координатах).
Ну или выбери специализированную книжку, чтобы основы линала пройти, так сильно быстрее будет, но беднее.
После этого можно уже читать по гладким многообразиям вступительные книжки, там в аппендиксах ещё кратко будут необходимые пререквизиты расписаны.
296 116356
>>6353
да, wolfram mathematica
297 116357
>>6348
Так там какие-то модулярные формы использовали и прочие сложные вещи о которых я знаю лишь по наслышке. И вроде как они зародились отдельно от алгебры, а из анализа с геометрией?
Типа, всякие идеалы колец и прочие вещи придумывали, чтобы ВТФ доказать, но в общем случае так и не получилось, лишь для конкретных $n$.
298 116358
>>6352
Группы Ли физикам очень нужны, конечно, но добавить туда их Винберг решил, думаю, не для физиков, а потому что он сам ими занимался как алгебраист.
Про группы в физике это маленький параграф всего.

>линал излагается с ориентацией на нужды физиков


Честно не вижу такого. Темы совершенно обычные, изложение и доказательства не координатные в основном. По-моему, книга написана в целом в очень алгебраичном стиле.
299 116359
>>29047 (OP)
Вижу, умненькие аноны собрались, и хочу задать, возможно, банальный вопрос.
Каким образом добираться до мотивировок введения тех или иных понятий, составляющих математику, тех или иных рассуждений, откуда черпать суть?
Вот скажем, натуральные числа - зачем они нужны? Зачем нужны множества? Думаю, задаваясь этим вопросом, можно лучше понять, что из себя они представляют, почему и зачем они такие.
300 116360
>>6359

>Думаю, задаваясь этим вопросом, можно лучше понять, что из себя они представляют, почему и зачем они такие.


нет, нельзя
чтобы лучше понять что-то, надо с этим работать, а не выбрасывать колебания воздуха в пустоту (или байты информации во всемирную сеть)
301 116361
>>6359

>Вот скажем, натуральные числа - зачем они нужны? Зачем нужны множества?


В древние века, когда палкой чертили круг и камешки на абаке складывали, зерно в мешках покупали и наследство делили.
302 116362
>>6359
Обычно после определений дают примеры. Возможно стоило бы делать наоборот. Чаще всего эти определения это выделение общего что есть в примерах. Либо просто способ назвать какую-то штуку так, чтобы под это название можно подвести все примеры.
Ты можешь жопой чуять, что арифметика остатков и повороты n-угольников почти что одно и тоже. Ты можешь жопой чуять, что арифметика матриц, арифметика целых чисел и арифметика многочленов тоже что-то общее имеет.
Любой человек интуитивно понимает, что такое предел последовательности. И определение предела это просто его описание, под которое можно подвести другие описания.
В большинстве случаев зачем нужно какое-то определение понятно из контекста, особенно начальные. Очень редко приходиться гуглить мотивацию.
303 116363
>>29047 (OP)
https://www.youtube.com/watch?v=UabGSrpEV5c
и это математик? я поражен. простота хуже воровства
304 116364
>>6363
это не математик. это Савватеев
305 116365
>>6363

>это математик?


Это экономист.
306 116366
>>6313
>>6320
Открыл Дьедонне прочитал по диагонале. Ужасная книга. Просто какой-то дроч аксиом вприсядку без результатов.
307 116367
>>6365
сорта говн
308 116368
>>6366
попробуй "Курс арифметики" Ж.-П. Серра
309 116370
>>6354
Бля, честно, боюсь браться за англоязычные книги, особенно по матану. Хотя, вроде, написано несложно, спасибо, сохраню
310 116372
>>6370

>особенно по матану


наоборот же. Тяжело читать худ книги, а такое легко идет. У меня А2 с натяжкой, никаких проблем с чтением на англ мат книг нет.
311 116373
>>6360

>чтобы лучше понять что-то, надо с этим работать


работать можно по-разному, поэтому интересно узнать не саму вещь, а ее предназначение
>>6361
сейчас мне нереально отправиться в прошлое
>>6362
предел довольно ясное понятие, как и связанные с ним метрика и топология, если я верно связал их с приближением, расстоянием и сужающимся кругом объектов.
наверное, есть способы помочь человеку почуять и интуитивно понять то или иное понятие.
контекст может помочь. значит, нужны книги с примерами. где найти именно такие книги?
19198099.jpg95 Кб, 634x879
312 116382
>>29047 (OP)
Аноны, поясните, на хуй математики нужны как класс, они же еще более бесполезные чем философы?
313 116383
>>6370
>>6372
Как показывает опыт, можно даже читать французские материалы по математике, не зная французский.
314 116384
>>6382
Тебя это ебать не должно.
315 116385
>>6384
Почему? Может я то же математик, а не бесполезный неудачник
316 116386
>>6382

>зачем нужны математики


не математика
317 116387
>>6382
как минимум, с ними можно обсудить математику и философию
318 116388
>>6386

>математика


Прикладная математика закончилась в 17 веке, потом начался треш, угар и шиза.
319 116389
>>6388
спасибо, ваше мнение очень важно и ценно для нас
320 116390
>>6372
Ну, попробую. В конце концов, ради этого я и учил английский
IMG20240615140031.png409 Кб, 513x572
321 116391
>>6389
Пожалуйста, обращайтесь
456456.png84 Кб, 1090x582
322 116392
>>6359
Существует такой предмет, как история математики. От Галилея и до середины 20 века все давно разобрано по полочкам, изучены все биографии, дневники, черновики, переписка и публикации не то что ведущих, но даже третьестепенных авторов. Но анализ исторической подоплеки это работа не на один год. И этим лучше заниматься ПОСЛЕ прохождения соответствующих курсов - чтобы понять суть срача вековой давности, нужно хотя бы в общих чертах знать его предмет.

>Зачем нужны множества?


Labyrinth of Thought: A History of Set Theory and Its Role in Modern Mathematics.
323 116393
>>6392
Спасибо за ответ.
Конечно, история касается моего вопроса, но можно и без истории.
Содержание самих понятий.
Про множества в курсе, нужно уместь брать сразу множество объектов, а не один отдельный, отсюда и множество. Это просто.
324 116394
>>6370
попробуй, я тоже студент-нематематик(инженигер), там базовые моменты в первых главах подробно расписаны и с мотивировкой, например только благодаря этой книге понял наконец какой смысл у матричного умножения, до неё для меня это правило "строка на столбец" было взятым с неба.
325 116395
Анон, хелп

Пытаюсь решить уравнение

$\cos x + \sin x + 2x = 2$

но вообще не могу понять куда двигаться? Перебрал все известные мне преобразования триг. формул, но упираюсь в тупик

Дайте совет какой-нибудь, пж
326 116396
>>6395
у этого уравнения один корень и получить его аналитически едва ли возможно
327 116397
>>29047 (OP)
https://arxiv.org/pdf/math/0211159
как мне быстро понять это. джайте роудмап что изучить.
328 116398
>>6397
начни с Ландау-Лифшица
329 116399
>>6398
сложно, но вместе с тем неподробно, галопом по европам
в принципе, совкодаун детективтся сразу рекомендацией этого говна (без обид)

это вообще кредо советской физмат школы: "сложно, но неподробно". удивительная способность идти сложно, но по верхам.

сравните это с учебными курами мит, гарварда, принстона. да блять, те же учебник терренса тао по матану в 10 раз лучше зорича объясняют, а он ведь даже не писатель
330 116400
>>6178

>math55


>Сжатый undergraduate курс, предназначенный для математически зрелых ребят. Какой смысл скакать галопом по Европам и изучать те же многообразия и дифференциальные формы, если ты не умеешь решать нестандартные задачи по обычному calculus из тех же листков 57 школы? Чтобы умных слов нахвататься?


о я как раз пост>>6399 написал и хотел добавить что math55b расхайпленный ололо хардкор(расхайпленные в западной физмат среде дрочеров на ХАРДКОРНЫЙ МАТАН уровне 17-20 летних реддиторов которые на elite STEM проги пускают слюни а не учатся матану) это ХУДШИЙ маткурс гарварда.

и он поразительно сильно напоминает ну например чистмат в матмехе спбгу. """хардкор""" ради хардкора, так сказать.

там нет ничего полезного для 95% ребят. 5% это хорошисты-отличниик, с охуенной базой, кучей свобоного времени и здоровой психикой, им МБ будет в пользу и не вред
331 116401
>>6399

>в принципе, совкодаун детективтся сразу рекомендацией этого говна (без обид)


пиздец, хотел потроллить тупостью, но внезапно двинулся в серьёзные щи

триггернуло, да? иди нахуй теперь
332 116402
>>6399

>мит, гарварда, принстона


Комиксодебил, плез
333 116403
>>6400

>там нет ничего полезного


Да. В курсе много лишнего, правильно начинать сразу с многообразий и дифф. форм
334 116412
>>6399

>сложно


???
335 116420
>>6382

>на хуй математики нужны как класс


Математики - не класс. Маркса читай.
336 116423
>>6420

>Математики - не класс


Класс бездельников и паразитов
337 116424
>>6423
Это ты с дворянством при абсолютизме перепутал.
338 116425
Какие листки лучше для вката в серьёзную математику с нуля — https://dev.mccme.ru/~merzon/v14/ или https://www.mccme.ru/free-books/57/davidovich.pdf ? Какие лучше развивают математическую культуру?
339 116429
>>6425
энциклопедия элементарной математики
340 116434
Ебать. Когда-то давно, читая MathOverflow StackExchange, я заприметил одного студента (у него была одна из самых высоких репутаций на сайте), тогда он показался мне крутованом и будущей звездой . Еще он вел блог про математику:

https://qchu.wordpress.com/

https://mathoverflow.net/users/290/qiaochu-yuan

Сегодня я что-то про него вспомнил и понял, что я давно о нем ничего не слышал. Погуглил и обнаружил, что он забросил PhD и стал "emotional coach":
https://gobeyondgoals.com/qiaochu-yuan/

вот это такой кринж: https://qchu.substack.com/p/5-tips-for-how-to-have-great-conversations

Пиздец. Что блядь пошло не так?
341 116435
>>6434

>Пиздец. Что блядь пошло не так?


Он понял, что занимался бесполезным говном.
342 116436
>>6434

> MathOverflow StackExchange



MathOverflow и StackExchange

Вот еще порция трэша:
https://www.dailymail.co.uk/news/article-9800091/Spoiled-rich-man-slammed-tweeting-resents-parents-giving-100k.html

Просто жесть.
343 116438
>>6434
Перегорел, иссушил психику. Я обычный троечник из фф урфу, но понимаю о чем речь.

Я считаю психотерапевты (не бабы, см последний пост Пикабу) нужны всему мфти и мм мгу. Хуже точно не будет, а мб будет плюс неск миллиардов к росту ввп России
344 116439
>>6438

>нужны всему мфти и мм мгу


Зочем? Рыночек все это говно порешает.
345 116440
Можете, пожалуйста, объяснить решение задачи? Если я уже месяц пытаюсь понять эту хуйню ебаную, значит ли это, что у меня умственная отсталость?
Не понимаю с первого же пункта. Типа почему там скорости складываются, с чего мы вообще делим расстояние между городами на какую-то скорость сближения и получаем время, через которое поезда встретились? Как эти пункты вообще связаны между собой? Пожалуйста, объясните тупой.
346 116441
>>6440

>Типа почему там скорости складываются, с чего мы вообще делим расстояние между городами на какую-то скорость сближения и получаем время, через которое поезда встретились?


Я тоже не понимаю с наскока, с чего они это взяли. Потому придётся решать самим.
Представь отрезок A__________B, если поезда выехали навстерчу друг-другу то они где-то встрется в точке C.
A___C_____B.
если $v_{1}$ скорость первого поезда, который из A вышел, а $v_{2}$ второго, и мы знаем что они встретились чреез 5ч, то первый прошел
$5v_{1}$ а второй $5v_{2}$, так же $5v_{1}=AC$, $5v_{2}=BC$, тк AC+CB=AB, то
$5v_{1}+5v_{2}=700$
$5(v_{1}+v_{2})=700$
$\frac{700}{v_{1}+v_{2}}=5$
но нам нужно найти скорости, потому лучше взять предыдущее выражение, а не дробь.
Если второй поезд выедет на 7ч раньше первого, и он встретится с первым через 2ч после того как выедет первый, то
$2v_{1}+(7+2)v_{2}=700$
$2v_{1}+9v_{2}=700$
составляй систему и решай
347 116442
>>6440
1. тебе следует разобраться, что такое скорость поезда. про сближение можешь не думать
2. сколько проехал каждый поезд и сколько они проехали вместе?
изображение.png15 Кб, 704x317
348 116448
>>29047 (OP)
Кому-нибудь здесь казалось, что ассоциативность, это слишком сложное свойство, чтобы быть аксиомой?
Представьте, допустим, операцию сложения как машину, куда мы вводим 2 числа и она выдает результат.
Тогда чтобы сложить 3 числа, нужно 2 машины, в первую ввести 2 числа и её выход подсоеденить к другой к 1 входу, и ввести там 3 число.
Так вот ассоциативность говорит, что мы можем поменять порядок машин, нижнюю поставить наверх, а верхнюю вниз. При этом видно, что числа $A,C$ остаются на месте, а вот что происходит с чилом $B$ непонятно в такой "модели".
349 116449
>>6448
Имею ввиду что если сравнить с той же коммутативностью, что говорит просто: поменяй числа местами результат тот же, ассоциативность выглядит нереально сложной. И у меня с самого вката остается ощущение, что есть некие более элементарные и простые свойства, из которых следует ассоциативность в привычном виде.
350 116451
>>6449

>И у меня с самого вката остается ощущение, что есть некие более элементарные и простые свойства, из которых следует ассоциативность в привычном виде.


у меня такое же ощущение. ассоциативность, сочетательность - означает, что можно рассмотреть последовательность, а при рассмотрении ее как вложенных пар расстановка скобок не имеет значения.

>При этом видно, что числа A,C остаются на месте, а вот что происходит с чилом B непонятно в такой "модели".


$A$ и $C$ остаются на месте в некоторых парах, в то время как $B$ размещается на разные места в паре.
351 116454
>>6448
Лучше представь, что тебе нужно распилить полено по отмеченным рискам. Вне зависимости от порядка распила кусков, результат распила будет одним и тем же. А коммутативность говорит о том, что результат распила безразличен к порядку, в котором будут разложены отпиленные куски. То есть ассоциативность утверждает эквивалентность всех порядков процесса, а коммутативность - эквивалентность всех порядков конечного результата. Как ни изощряйся с пилой, в итоге получишь один и тот же набор кусков. Как ни изощряйся с раскладкой кусков, в конце их все равно придется сложить в мешок.

Ну то есть речь идет о порядке над множеством операций и порядке над множеством элементов, участвующих в этим операциях. Аксиомы утверждают безразличность к порядкам как в первом, так и втором случае. Это максимально естественные интуиции, берущие начало в манипуляциях с однородными предметами или веществом.
352 116455
>>6449
связываются две структуры: последовательность и двоичное дерево
353 116456
>>6454

>нужно распилить полено по отмеченным рискам


можно представить, что полено коническое, сужается к одному концу. тогда будет ясно различие ассоциативности и коммутативности
354 116465
>>6434

>Что блядь пошло не так?


Юдковский и Дуыыцкщтп
355 116466
>>6465
*lesswrong
356 116469
>>6425
Бамп вопросу.
357 116471
>>6425
>>6469

Откуда взялся этот дроч на листки 57ой школы? Ведь листки сами по себе не превратят вас в пятисемита. Матшкола - это не только подборки задач, но и общение со сверстниками со сходными интересами, обратная связь от преподавателей. Другой момент, это то, что 57ая была крутой во времена Совка, когда на занятия приглашали первоклассных математиков, и они рассказывали школьникам сюжеты из продвинутой математики. Кстати, при всей своей крутости, никаких великих математиков 57ая не породила. Что до листков, с которыми вы постоянно носитесь, этот материал покрывается первым курсом в сколько-нибудь приличном университете. Сколько хорошо написанной литературы для студентов и школьников - нет, не хочу, хочу листки.
image.png95 Кб, 573x362
358 116472
>>6425
Я не уверен, что так уж полезно вкатываться в математику по листкам в отсутствии преподавателей.
359 116473
>>6471
Хованов, Бондал, Кузнецов, Капранов.
360 116475
Гротендик в детстве виноград собирал, а не листки решал и олимпиады дрочил. Думайте.
361 116476
>>6475
ты путаешь достаточное условие и необходимое

может, тебе стоит листки порешать
19198099.jpg95 Кб, 634x879
362 116478
>>6475

>Гротендик


Кто такой, чем знаменит?
363 116480
>>6478
Высрал хуйню про топологию и подох, типичный бесполезный маняматик
364 116482
>>6478
Большим членом
365 116484
>>6471
В книжке давидовича(того самого педофила) в списке учеников класса, при работе с которым был написан сборник, есть Александр Ефимов(EMS, конгресс), Павел Плесков(топ-2 kaggle), какой-то чувак Оскар выиграл в качестве не-помню-кого в Disney, почти все остальные - программисты в бигтехе. Так что дрочить есть на что(хотя бы приподняться мпх может), кмк problem-solving скилл эта макулатура нормально ставит, хотя может английские intro to proof книжки типа How to Think like a mathematician, Лары Алкок и прочие делают это лучше. Ну типа, полноценно потратить полгода на листки - это как бросить неумеющего плавать ребенка в море. Если способный, то научится доказывать и дальше можно спокойненько читать хорошие англоязычные учебники, а без навыка доказательства челик может и умереть, даже на хорошей книжечке.
366 116485
>>6482

>членом


Многочленом?
367 116490
Можете, пожалуйста, объяснить, что он сделал в самом начале? И почему нельзя решить так, как решила я?
368 116493
>>6490
на первом пике использованы основное триг. тождество и формула для синуса двойного угла. на втором пике написано слишком неясно, чтобы я понял, что пытался сказать автор_ка
369 116494
>>6490
основное тригонометрическое тождество и квадрат разности
Peter-Griffin-vomit-meme-9.jpg38 Кб, 600x400
370 116496
>>6484

>программисты в бигтехе


>problem-solving


>intro to proof книжки


>книжечке

image.png19 Кб, 713x530
371 116499
дана окружность с известным радиусом. понятна, что для точки 0,0, длина вектора до любой точки на границе окружности равна радиусу. а как найти эту длину для произвольной точки внутри окружности с произвольным углом?
372 116500
>>6499
Записать в виде $\vec{\rho}=\vec{R}-\vec{r}_0$, где $\vec{\rho}$ — вектор от точки $(x_0, y_0)$ до окружности, который нужно найти, $\vec{r}_0=(x_0, y_0)$, а $\vec{R}=(\cos{\theta}, \sin{\theta})$ — вектор от начала координат до окружности. Дальше просто нужно найти длину вектора $\vec{\rho}$
373 116502
>>6500
*$\vec{R}=(R\cos{\theta}, R\sin{\theta})$
374 116504
математика это самый скучный язык в мире
375 116505
>>6504

>язык


Это не язык
376 116507
>>6504
А ты няшный?
377 116508
>>6507
а что тебе даст ответ на вопрос?
378 116509
Где найти студента или преподавателя, которому можно будет сдавать на проверку листки из тривиума Вербицкого и Каледина? Я конечно понимаю, что сейчас век капитализма, но в НМУ преподаватели всё делают бесплатно, где таких искать? Сразу в НМУ обращаться не хочу, потому что чувствую, что не хватает зрелости.
379 116510
>>6509
едва ли кто будет этим заниматься с тобой бесплатно

можешь попробовать поискать среди студентов матфака, кто подрабатывает репетирством.
380 116511
>>6510
И что тогда делать? Окромя бочки.
381 116512
>>6511
забить на свои комплексы и идти в НМУ
382 116513
>>6510

>едва ли кто будет этим заниматься с тобой бесплатно


Единственный способ для математика заработать на кусок хлеба, а этот хуй платить не хочет.
А если вдруг найдется такой, надо ломом этого штрейкбрехера отпиздить.
383 116514
>>6509
Там элементарные задачи.
image.png4 Кб, 338x64
384 116515
>>6500
угадал?
385 116516
>>6515
блин не сработала по ходу математика неточная наука
386 116517
Подскажите, пожалуйста, какой сборник задач подходит к учебнику Винберга по алгебре?
387 116518
Нет же ничего зазорного в том, чтобы иногда подсматривать в решебник какого-нибудь задачника? Ну типа зная решение, в следующую похожую задачку уже не будет таких сложностей.
image.png139 Кб, 1237x821
388 116519
и так тоже не сработало. Жаль, что мир оказался непознаваемым, не хотелось в это верить.

Очевидно, что человек не был создан для понимания этого мира. Весь заложенный в нём интеллектуальный потенциал - это минимальный набор, необходимый для выживания до периода создания потомства. После этого, смысл человеческого существа перестаёт иметь значение, и ему остаётся только кануть в ту бездну из которой он выполз
389 116520
>>6516
Что не сработало?
image.png141 Кб, 1409x783
390 116521
391 116522
>>6519
Вместо $(R-x_0)\cos(a)$ должно быть $R\cos(a)-x_0$.
То же самое со второй координатой.
Вместо $v_{2x}=[x_0, v_2[1]]$ должно быть $[x_0, v_2[1]+x_0]$.
То же самое с $v_{2y}$.

https://www.desmos.com/calculator/gpbfprhzbc
392 116523
>>6517
Можно, наверное, упражнения из Алгебы Ленга решать из схожих параграфов. Но там вроде бы ответов нет, так что хз.
image.png145 Кб, 1636x816
393 116524
>>6522
офигеть! оказывается мир познаваем, антропоморфичен, гармоничен
394 116526
>>6517
Кострикин
395 116529
>>6524
Ты мог нарисовать просто ось координат, окружность и провести эту палку там, где тебе можно. Ну математики, ну тупые.
396 116530
>>6529
где тебе нужно*
397 116531
>>6529
ты предложил действительно очень интересную идею, но стирать и заново отрисовывать всё это каждый раз после смещения координат x_0 и y_0 будет не очень быстро (даже таким удобным инструментом, как paint)
2024-07-17 173335-NVIDIA GeForce Overlay DT.jpg57 Кб, 896x169
398 116532
Как вам такая система оценки на экзаменах?
399 116533
>>6532
Это где такое говнище?
400 116534
>>6533
поступление в магу РЭШ

контекст: там рейтинг сдающих, и чем ты выше - чем больше скидка(вплоть до 100%) на обучение. т.е. есть стимул решать все, а стимула осторожноичать и не обжечься нет
401 116535
Как лучше изучать предметы, где много доказательств?
Например, я понимаю теоремы, понимаю доказательства. Через пару дней попроси меня доказать что-нибудь и вряд ли я смогу повторить без подглядывания. А через какое то время и сами теоремы забываются. Например я помню про что такая-то теорема, но дать точную формулировку не смогу.
Вообще нужно ли как то вбивать в голову доказательства? Или это нужно скорее тем кто работает преподом, ну и естественно эти доказательства за годы работы у них запоминаются.
402 116536
>>6535

>Вообще нужно ли как то вбивать в голову доказательства?


нет, не нужно
нужно решать задачи/доказывать самому
403 116538
>>6535
Теоремы — это высказывания о свойствах каких-то математических объектов чаще всего. Тебе нужно понимать, что это за свойства и почему они есть именно у этого объекта, знать примеры и контрпримеры и понимать, почему они таковыми являются. Очень часто, исходя из этого, доказательство само запомнится. Но если оно техническое, то особого смысла его запоминать нет.
404 116539
>>6538
А какие доказательства называются техническими?
405 116540
>>6539
это петух-неосилятор написал, он сам не понимает, что несёт
406 116541
>>6539
Имо, теоремы о существовании каких-нибудь бесконечных объектов, доказательства которых опираются на условную лемму Цорна, например, никаких инсайтов об этих объектах не дают и (почти) никакого смысла их запоминать нет. Доказательство теоремы о существовании алгебраического замыкания или о K-инъективной резольвенте в категории комплексов пучков модулей ничего полезного об этих сущностях тебе не расскажут.
Сюда же относятся теоремы, ради которых по десятку технических лемм предварительно доказывают. Многие из этих технических лемм больше никогда тебе не встретятся. Но, возможно, если ты хочешь стать аналитиком, то есть смысл их учить и запоминать.
>>6540
Ёбырь твой?
407 116543
Ну чё, сколько решили бы, математики?
https://www.imo2024.uk/s/IMO-2024-Paper-1-Solutions.pdf
408 116544
>>6543
Изично всё решеается через преобразование фурье.
409 116545
>>6541
ну ладно, обознался
буду аккуратнее
ты всё равно накатал что-то бессодержательное, хоть и с пучками
410 116547
>>6541

>Ёбырь твой?


Виноват. Так чмонделю сральню разворотил что он до сих пор никак не подошьет. Так и бегает со свое разорванной сральней по всему разделу и трясет ей перед всеми.
411 116548
>>6545
Хз, простая мысль: если доказательство теоремы "конструктивно" (взяли и построили объект или взяли и разложили его как-нибудь), то оно опирается на какие-то свойства объектов, фигурирующих в условии теоремы, которые можно "пощупать" на примерах, а на контрпримерах обнаружить препятствия к утверждению. Тогда понимание этого объекта = умение доказать теорему самостоятельно.
412 116549
>>6548
это всё замечательно, только на практике совершенно бесполезно
это подобно рассуждениям ромы про метаязыки и цветные библиотеки - чудесно, прекрасно, только что с этим глубоким знанием теперь делать и какой вообще в этом смысл? он появляется, возможно, когда ты всё это сам у себя в голове устроил и наделил собственными ассоциациями

но до того - это просто бессмыслица

А доказательства, которые опираются на лемму Цорна, обычно рассказывают про саму лемму Цорна
413 116550
>>29047 (OP)
Это ошибка на сайте или я туплю?
414 116551
>>6550
Ты тупишь
415 116552
>>6549
Ну я не про какие-то высокие материи говорю и эзотерическое прозрение, а как раз про практику.
416 116553
>>6550

>Это ошибка на сайте или я туплю?


твоя ошибка
$m \cdot (1 + 80 \%) \cdot (1 - x) = m$
$1.8 \cdot (1 - x) = 1$
$\frac{9}{5} \cdot (1 - x) = 1$
$1 - x = \frac{5}{9}$
$1 - frac{5}{9} = x$
$x = frac{4}{9}$
417 116554
>>6553
$1 - \frac{5}{9} = x$
$x = \frac{4}{9}$
418 116555
>>6553
Спасибо, анон.
Я действительно недокрутил решение.
decreased by - уменьшилось на.
(1 - x) нужен был, как у тебя.
>>6551
Спасибо.

Как же математика мозг выносит, и это блядь начальная школа.
419 116556
>>6541

>Доказательство теоремы о существовании алгебраического замыкания


Априори не очевидно, что алг. замыкание существует, тем более для любого поля.
Более того, доказательство можно сформудировать так, чтобы там неконструктивен был только один, не самый важный, шаг.
Сначала мы показываем, что у каждого поля $K$ существует кольцо разложения всех многочленов $K[x]\setminus \{0\}$ (т.е. свободная $K$-алгебра, в которой все многочлены над $K$ разлагаются), которое нетривиально. Доказательство нетривиальности как раз самый важный и сложный шаг доказательства. Потом мы строим факторкольцо по максимальному идеалу, которое будет полем - использование Цорна сводится к этому шагу. Показываем, что это факторкольцо это алг. замыкание $K$.
420 116557
>>6556
Я не говорил, что теорема неважна. Я говорил, что помнить её доказательство смысла нет.
421 116558
>>6557
Я спорю с утверждением, что доказательство существования алг. замыкания поля ничего не говорит об алг. замыкании или о полях. Как минимум в той форме, которую я привел, доказательство дает достаточно материала для "прощупывания" и понимания, что может пойти не так в случае общих коммутативных колец. В целом, тот факт, что нет тривиального обобщения алг. замыкания поля на случай комм. колец, уже должен говорить, что теорема не просто "техническая".
422 116559
>>6558
Окей, согласен, плохой пример.
423 116561
>>6441
Ой, спасибо тебе большое, я все наконец-то поняла
424 116562
Я поняла, как он преобразовал выражение под корнем, как у него получился модуль и как он раскрыл его. Только вопрос - откуда взялось п/4, как понять, что именно там сначала одно будет больше, а потом другое?

И еще. Зачем он добавил ко второму 2 корня из двух?
425 116563
>>6562
По первому вопросу: посмотри, где растёт/убывает синус, где убывает/растёт косинус. В точке $\pi/4$ они равны. Что происходит до этой точки и после неё?
426 116564
>>6562
если ты понял_а, как раскрывается модуль, то не должно быть вопроса про пи/4: оно из раскрытия модуля и получается там выражение под интегралом приводится к одному синусу, после чего всё проще

к произвольной константе можно прибавлять любую другую фиксированную константу, даже $2\sqrt{2}$ (константа всё равно останется произвольной), потому формально ошибки нет, хотя смысл ускользает

вообще, что тебе мешает провести все вычисления самостоятельно, если идея, как делать, уже понятна
1702560121savvateevav.webp30 Кб, 760x506
427 116565
Мнение двача о пикрил?
428 116566
>>6565
дрыщ скелететон мало качается мало каши ест
429 116567
>>6565
хуйня перехайпленная
объясняет хуево, подача материала никакая, с таким же успехом можно книгу читать и будет намного эффективнее чем его видео
в одной говногруппе даже разыгрывают его говнокниги с автографами. ебало имажине?
ладно бы у него была премия филдса или какие то достижения в науках, но это буквально ютуб клоун ничего из себя не представляющий
430 116568
>>6565
Какую бы я тему у него не смотрел, всегда я нихуя не понимал. Чел слово в слово пересказывает учебники. Не знаю, понимает ли он что-то. Похоже что нет, обычный заложник китайской комнаты.
431 116569
>>6565
Какую бы я тему у него не смотрел, никогда не мог отличить от русского националиста. Чел слово в слово пересказывает методичку. Не знаю, понимает ли он что-то. Похоже что нет, обычный заложник пропаганды.
432 116570
>>6563
>>6564
С п/4 поняла. Можете поподробней про 2 корня из двух объяснить?
433 116571
>>6570
там написано $C+2\sqrt{2}$, где под $C$ понимается произвольная константа. в этом $2\sqrt{2}$ нет никакого смысла
434 116572
>>6571
>>6570
$2\sqrt{2}$ нужно, чтобы функция непрерывной была в точке $\pi/4$.
435 116573
>>6572
*функции из множества первообразных
436 116574
>>6572
в самом из деле
я и не заметил
изображение.png326 Кб, 1178x849
437 116575
Помогите мне понять обведённое красным.
438 116576
>>6575
Определение синусов/косинусов через треугольники знаешь? Если да, то погугли сам лучше, посмотри картинки, так понятнее будет. Начни с полярных координат, потом уже сферические.
439 116577
>>29047 (OP)
есть книга или справочник с описанием и примерами всех маняматических символов?
441 116579
>>6578

>Википедия


лол
442 116580
>>6579
В чём лол?
443 116581
>>6580
в ненадежности
444 116582
>>6581
Сомневаешься -> переходишь по ссылке на источник.
Математическая википедия неплохо написана, профессиональные математики ей тоже пользуются.
445 116612
В этом https://old.mccme.ru//ium//f04/experimental.html курсе в качестве предпосылки указывается знание теории множеств и логики. Откуда можно всё это подчерпнуть? Запросы такие:
1) Узнать правила логики, чтобы при решении задач не допускать логических ошибок.
2) Научиться решать задачи, опираясь на определения и выше указанные правила, а не слепую слепую интуицию и аналогии (я знаю, что чтобы научиться решать задачи, нужно их решать, но хочется сделать этот процесс более правильным).
3) Узнать основы теории множеств и математической логики (программа указана внизу страницы курса), разобраться с тем, как работает аксиоматический метод.
446 116613
Хочу пройти школьный курс алгебры и геометрии для подготовки к егэ. Есть ли разница между авторами или можно брать любых ? Мордкович, Макарычев и т.п.

У меня есть еще второй вариант взять чисто курсы подготовки к егэ, но мне такой вариант не очень нравится.
447 116614
>>6612

>НМУ


>вербит


>тривиум


Сразу нахуй.

Сука, как же меня заебала это постсоветская клоунада. Во всех нормальных странах вкат начинается с транзишена, и только в сраной все происходит через пионерские зарницы, разрывы сраки и драки анальными костылями.

Чел, тебе не нужны альтернативно одаренные программы, созданные советскими фриками и аутистами для таких же фриков и аутистов. Тебе нужен курс по пруфам от психически здоровых людей для психически здоровых людей. Бери последние издания A Transition to Advanced Mathematics, или How to read and do proofs.
448 116615
Утверждается, что каждое множество, содержащее все свои предельные точки, имеет конечное покрытие. Но числовая прямая содержит все свои предельные точки, однако все её покрытия бесконечны. Или я не прав?
449 116616
>>6612

> в качестве предпосылки указывается знание теории множеств и логики


скорее всего, можно удовлетвориться наивным подходом. если слово "множество" само по себе вызывает оторопь, то в этот тривиум лучше не лезть, я подозреваю

>>6613
егэ - это огромный набор однотипных задач
их лучше всего осваивать по специализированным задачникам и экзаменам прошлых лет
450 116617
>>6615
всякое множество можно покрыть им самим (одним)
сформулируй точнее, что ты имеешь в виду
451 116618
>>6615

>Утверждается, что каждое множество, содержащее все свои предельные точки, имеет конечное покрытиее.


Это неверно, множество должно быть замкнутым (т.е. содержать все свои предельные точки) и в дополнение к этому ограниченным.
Screenshot2024-07-22-21-13-29-952com.google.android.apps.docs-edit.jpg156 Кб, 1080x322
452 116619
>>6618
На пикче говорится, что достаточно. Или нет?

>>6617
В тех книжках, что я читал знак включения множества в его покрытие строгий. Не ошибка-ли если множество совпадает со своим покрытием?
453 116620
>>6619
на пикче утверждение правильное: обрати внимание, в чём отличия от твоего первоначального вопроса
454 116621
>>6620
Я уже заметил, да. Но у меня же более сильная формулировка. Что не так с требованием содержать все предельные точки?
455 116622
>>6621
на пикче, про предельные точки чего именно идёт речь?
456 116623
>>6622
Последовательностей точек множества, я про то же говорил.
457 116624
>>6623

>Последовательностей точек множества


правильно

но здесь>>6615 ты говорил про другое - про предельные точки самого множества

почему прямая не удовлетворяет пункту а)?
458 116625
>>6624

>предельные точки самого множества


А это не тоже самое? Я видимо совсем ньюфаг.

>почему прямая не удовлетворяет пункту а)?


Есть расходящиеся последовательности?
459 116626
>>6619
>>6625
Посмотри на последовательность $\{1,2,3,4,\dots\}$ в $\mathbb{R}$.
460 116627
>>6625

>А это не тоже самое?


есть $X \subset M$ - множество, взяли $\mathcal X = \{x_i\} \subset X$ - последовательность в $X$. предельная точка последовательности $\mathcal X$ - не то же самое, что предельная точка объемлющего множества $X$: она предельная по отношению именно к $\mathcal X$. так же предельная точка $X$ - не то же самое, что предельная точка пространства $M$. проверь определение

>Есть расходящиеся последовательности?


у которых нет предела: например, у последовательности натуральных чисел $\mathbb N$ нет предельной точки (относительно стандартной метрики в $\mathbb R$)
461 116628
>>6615
compact(пункт a) <=> closed & bounded <=> every open cover has a finite subcover(пункт в)
Не из каждого покрытия R можно выделить конечное подпокрытие: например, если в качестве покрытия R взять окрестности длины 1 точек из R, конечного подпокрытия выделить не получится.
462 116629
>>6626
>>6627
Спасибо. Меня смутило, что в этой книжке обычно под последовательностью понимается последовательность Коши. Вроде разобрался. Про предельные точки почитаю.
463 116630
>>6628
Спасибо, про третье условие не слышал.
464 116631
>>6629
последовательность в сущности - любое (не более чем) счётное множество
465 116632
>>6631
А несчетных быть не может? Никогда не интересовался.
466 116633
>>6632
для несчётных используется термин направленность но это уже более-менее экзотика
467 116634
>>6633
Спасибо
468 116635
Хочу решить задачу тысячелетия, получить лям баксов и мировую известность. С чего начать?
469 116636
>>6635
С Африметики.
470 116637
какие учебники лучше зорича?можно англ, real anal(isis)

рудина знаю, 1 в1 зоричем нудное говнго. мне нужно больше, глубже, разжеваннее подробнее и с бОльшего кол-васторон, пустьи будет в 3-5 раза объемнее зорича, с геометрией в иедалечтобы были красивые иллюстрации в стиле 1blue3bron мне через геометричсекие образы матан понимать гораздо легче чем формулами на бумаги.
471 116641
>>6637

>, пустьи будет в 3-5 раза объемнее зорича,


4-х томнит Решетняка.
472 116642
>>6641
*томник
474 116646
>>6637
анализ на R: Abbott
многомерный вплоть до форм и т.Стокса: Ted Shifrin, Hubbard&Hubbard

Мотивировки, картинки, байки, полные solution manuals, современные обкатанные годами курсы от педагогов, к шифрину есть лекции на ютубе
475 116647
>>6646

>современные обкатанные годами курсы от педагогов


Англопидарасня в матешу может?
476 116648
>>6646
о, глянем, только давай конкретнее с названием
477 116649
>>6612
как правила, так и ошибки ты почерпнешь в жизни. ошибаться нормально. поэтому тебе вряд ли понадобится книга по наивной логике.
обсуждение вариантов, как интуиционизм, конструктивизм, формализм, можно найти в разных книгах. тебе они тоже не нужны. за исключением формализма, по желанию. См. Haskell B. Curry, Foundations of Mathematical Logic, я имел дело со 2-м изданием.
Также есть русский перевод 1-го издания, в нем больше содержательных ошибок. Читая его желательно сверяться с др. изданием.
image.png51 Кб, 1700x414
478 116650
Интересная задача, есть идеи какие-нибудь? Может быть подскажете какая теорема тут поможет?
479 116651
>>6650
Ряды Тейлора проходил?
480 116654
>>6651
да, но как они тут помогут, не могу сообразить. ведь если составить ряд с $a_n = c_n \cdot x^n $ то он не обязательно будет сходиться в точке отличной от нуля
481 116655
>>6654
во-первых, обрати внимание, что производные функции $\sum c_n \cdot x^n $ в нуле не равны $c_n$; надо брать $c_n/n!$

во-вторых, надо умножить каждое слагаемое на подходящую срезающую функцию $\chi_n$ (бесконечно-дифференцируема, с компактным носителем, равна $1$ в окрестности нуля и $0$ в немного большей окрестности, носитель сужается достаточно быстро с ростом $n$)

'это довольно стандартный трюк в анализе на самом деле
482 116656
>>6654
Там часто в качестве примера гладкой-неаналитической приводят функцию в виде экспоненты от чего-то там. С помощью неё можно собрать т.н. функцию-"шапочку", она будет гладкой везде, равной константе в некоторой окрестности нуля и не равной нулю только в некоторой большей окрестности. Такой тип функций называется bump functions, хз как на русском.
Мне кажется, домножая правильным образом члены прото-ряда на такие функции, можно добиться хорошего поведения ряда везде, так как эта функция всё зануляет за пределами своего носителя, а условие в задаче у тебя только требования в окрестности нуля предъявляет, где такая функция просто 1 равна. Правда нужно ещё проверить, наверное, как производные ведут и подобрать какие-нибудь масштабирующие коэффициенты, чтобы оценку сделать для сходимости. Выглядит, как довольно большое количество работы.
Пыня.jpg44 Кб, 1280x720
ПУЧК 483 116662
ПУЧК ПУЧК ПУЧК ПУЧК ПУЧК ПУЧК ПУЧК ПУЧК ПУЧК ПУЧК ПУЧК ПУЧК ПУЧК ПУЧК ПУЧК ПУЧК ПУЧК
484 116663
Помогите найти закономерность:

AAXXBAAABXXBBXXXBBXXX

Где X - A, B или C.
485 116664
$\neq$ (тест)
image.png216 Кб, 1108x909
486 116665
>>6663
если A - это 0, B - 1, то
487 116666
Посоветуйте задачники (можно на английском) по следующим предметам:
действительный анализ функций нескольких переменных
обыкновенные дифф. уравнения
комплексный анализ
теория меры

Спасибо.
488 116667
>>6662
ГРООТ ГРОТ ГРОТ
изображение.png131 Кб, 986x718
489 116668
Хотел спросить, почему данный ряд нельзя просто разложить на бесконечно малые? Из [math]\sqrt{n-1}, \sqrt{n+1}[/math] вынести [math]\sqrt{n}[/math], приведя числитель к виду [math]\sqrt{n}\cdot (\sqrt{1+\frac{1}{n}}-\sqrt{1-\frac{1}{n}}) \sim \sqrt{n}\cdot ({1+\frac{1}{2n}}-{1+\frac{1}{2n}}) = \sqrt{n}\cdot(\frac{1}{n}) = \frac{1}{\sqrt{n}}[/math]
Выглядит легче, как мне кажется, но эту тему в ВУЗе пока ещё не проходил - интересно стало, почему в рядах так нельзя делать?
490 116669
>>6668
ты написал правильную выкладку, правда что такое « разложить на бесконечно малые» мне не очень понятно
491 116670
>>6666
сотона ебаная, нахуй тебе задачники? пиздуй демонов в аду пинать, чтобы работали лучше
492 116671
>>6670
бро, не круто. Больше работающих демонов - больше зла и насилия в этом, и так не в самом лучшем мире
493 116672
верно ли что любая функция расладывается на линейные множители? а если нет, как это доказывается/опровергается для произвольной функции?
494 116673
>>6655
Даже если мы найдём такую срезающую функцию, то мы получим, что ряд будет сходиться поточечно, а следует ли из этого, что предельная функция будет бесконечно гладкой?
495 116674
>>6656
может пойдёт функция $ f = e^{-\frac{1}{x(x-1/n)}} $ если $ x > 0 , \, \, $ $0$ если $ x = 0, |x| \geq1/n , \, \, \,e^{\frac{1}{x(x+1/n)}} $ при $x <0$ у нее все производные в 0 равны 0, она бесконечно гладкая. Может надо вычесть её из членов ряда с подходящим коэффициентом, чтобы была сходимость? Я нигде не нашёл решения этой задачи, даже статьи на вики нет с этой теоремой.
image.png118 Кб, 1918x766
496 116675
>>6674
UPD ошибка, при $ x > 0 , f = e^{\frac{1}{x(x-1/n)}} $
497 116676
>>6674
Сто пудов есть на mathstackexchange, в англовики я вроде тоже встречал.
498 116679
Эх, вот бы умненького кунчика-математика, чтобы вместе сидеть дома за учебниками 🥰
499 116680
>>6673
пусть $\varphi$ гладкая функция, для которой $\varphi(x) = 1$ в окрестности $0$ и $\varphi(x) = 0$ для $|x| \geq 1$.

рассмотрим функцию $g_n(x) = \frac{c_n}{n!}(\varepsilon_n x)^n\varphi(\varepsilon_n^{-1} x)$, где числа $0 < \varepsilon_n < 1$ будут определены ниже.

читатель легко увидит, что для каждого $k < n$ имеет место оценка
$|g_n^{(k)}(x)| \leq C_{n,k} \varepsilon_n^{n-k}$ с некоторой положительной константы $C_{n,k}$, зависящей только от $n$ и $k$.

выбирая $\varepsilon_n$ настолько маленькой, чтобы $C_n \varepsilon_n^{-k} < 2^{-k}$, мы обнаруживаем, что ряд $f(x) = \sum_n g_n(x)$ равномерно сходится для всех $x$, ровно как и сходятся равномерно все ряды, полученные из него почленным дифференцированием.

остаётся заметить, что $f^{(n)} = c_n$, что и требовалось
500 116681
>>6680
поправки:

>выбирая $\varepsilon_n$ настолько маленькой, чтобы $C_{n,k} \varepsilon_n^{n-k} < 2^{-k}$



>остаётся заметить, что $f^{(n)}(0) = c_n$

501 116682
Если Х - какое-то множество или объект категории, то Aut(Х) - группа. Но ведь мы тогда можем продолжить это в ряд X -> Aut(X) -> Aut(Aut(X)) -> ... Что-то я нигде такого не встречал, то есть этот ряд не очень полезен. Почему?
Обновить тред
Двач.hk не отвечает.
Вы видите копию треда, сохраненную вчера в 09:45.

Скачать тред: только с превью, с превью и прикрепленными файлами.
Второй вариант может долго скачиваться. Файлы будут только в живых или недавно утонувших тредах. Подробнее

Если вам полезен архив М.Двача, пожертвуйте на оплату сервера.
« /math/В начало тредаВеб-версияНастройки
/a//b//mu//s//vg/Все доски